Tải bản đầy đủ (.pdf) (66 trang)

Tạp chí Toán học tuổi thơ kỳ số 137 và 138

Bạn đang xem bản rút gọn của tài liệu. Xem và tải ngay bản đầy đủ của tài liệu tại đây (5.54 MB, 66 trang )

<span class='text_page_counter'>(1)</span><div class='page_container' data-page=1></div>
<span class='text_page_counter'>(2)</span><div class='page_container' data-page=2></div>
<span class='text_page_counter'>(3)</span><div class='page_container' data-page=3>

2


Nh×n ra thế giới



Đề thi Olympic Toán học trẻ Quốc tế


Bulgaria (BIMC 2013)



DTH



Kết quả Thi giải toán qua th


Phá án cùng thám tử Sêlôccôc


Ai là kẻ khả nghi?



Nguyễn Tuấn Anh



Đến với tiếng Hán



Bài 52:

Bà nội đi bộ trong công viên



Nguyễn Vũ Loan



Học Vật lí b»ng TiÕng Anh


Unit 10.

Transfer of heat



Vị Kim Thđy



Bạn đọc phát hiện


Giải một bài toán số học



ậộo Huy Trđêng



Thịch ệÊu! Thịch ệÊu ệẹy!



TrẺn ệÊu thụ mét trẽm mđêi tịm


Dộnh cho cịc nhộ toịn hảc nhá


Mẻ réng hai ệỊ toịn hay



TrÇn Quang Hùng



Chữ và chữ số


Kì 13



Trng Cng Thnh



Cuc thi dnh cho các thầy cô giáo


Thi ra đề kiểm tra, đề thi tốn



ậỊ kiĨm tra chđểng I ậỰi sè 7


Lỡch sỏ Toịn hảc



Sù ra ệêi cựa phđểng trừnh bẺc hai, bẺc ba


vộ bc bốn



Hoàng Trọng Hảo



Trò chuyện


Cây khế



Nguyễn Đức Quang



Trang thơ



Măng Đen

(Vũ Nam Trực)




Mùa vàng

(Cao Ngọc Toản)



Vo thm vờn Anh


Dch thạ nộo?



Hă Thỡ Thu Hđêng



Trđêng Olympic


ậĨ ệi nđắc ngoội



Vị Kim Thủy



Rubic Hỏi... Đáp



Đề thi giải toán qua th


Tr 34


Tr 38


Tr 42


Tr 44


Tr 45


Tr 46


Tr 58


Tr 60


Tr 61


Tr 62


Tr 63


Tr 64


Tr 47


Tr 48



Tr 52


Tr 56


Tr 51


TRONG SỐ NÀY



Từ năm 2010 đến nay, chi phí sản xuất
như giá giấy, cơng in, cước vận chuyển,
kho bãi,... liên tục tăng. Được phép của
Nhà xuất bản Giáo dục Việt Nam, Toán
Tuổi thơ điều chỉnh giá bìa của Tạp chí
từ năm học 2014 - 2015 như sau:
Tạp chí Tốn Tuổi thơ 1 - Dành cho
Tiểu học


- Giá bán lẻ số gộp tháng 7+8/2014
(TTT1 số 165+166) là 16000 đồng.
- Từ tháng 9 năm 2014 giá bán lẻ mỗi
tờ là 10000 đồng.


Tạp chí Tốn Tuổi thơ 2 - Dành cho
THCS


- Giá bán lẻ số gộp tháng 7+8/2014
(TTT2 số 137+138) là 16000 đồng.
- Từ tháng 9 năm 2014 giá bán lẻ mỗi
tờ là 10000 đồng.


Tạp chí mong tiếp tục nhận được sự hợp
tác của các Quý đơn vị và bạn đọc trong
cả nước.



Trân trọng.


TTT


</div>
<span class='text_page_counter'>(4)</span><div class='page_container' data-page=4></div>
<span class='text_page_counter'>(5)</span><div class='page_container' data-page=5></div>
<span class='text_page_counter'>(6)</span><div class='page_container' data-page=6></div>
<span class='text_page_counter'>(7)</span><div class='page_container' data-page=7></div>
<span class='text_page_counter'>(8)</span><div class='page_container' data-page=8></div>
<span class='text_page_counter'>(9)</span><div class='page_container' data-page=9>

8



Kính thða các vị đại biểu, quý thầy cơ giáo, các phóng viên báo đài, các bạn tình nguyện


viên và các em học sinh thân mến.



Thêi gian ệở biạn mét em hảc sinh lắp 5 thi Toịn Tuữi thể lẵn ệẵu tiến tỰi Nam ậỡnh 2005


thộnh sinh viến nẽm thụ 2. Thạ lộ ệở ệạn Olympic lẵn thụ 10. Con tộu Olympic Toịn Tuữi


thể chẻ ệẵy tri thục di chuyÓn tõ Nam ậỡnh, qua Quờng Ninh, Hời Phưng. Ba ga ệẵu tiến


Êy mẫi chẳng cịc bỰn thÝ sinh gẳp 6 bội toịn khã vắi thêi gian giời bội 90 phót. Nẽm 2008


tộu tắi Hộ Néi. Bội thi cị nhẹn tẽng lến 14 bội toịn vộ thếm cuéc thi Tiạp sục ệăng ệéi về


cỉng bì ngì vắi nhiỊu ệéi. Còng tõ ệã mẫi toa tộu chử chẻ 6 thÝ sinh. Nẽm 2009 tộu mỰnh


dỰn chỰy vộo Thõa Thiến - Huạ. Tõ ệẹy ệoộn tộu mang tến Olympic Toịn Tuữi thể. Song


hộnh hai cuéc thi cho TH vộ THCS ệem lỰi sinh khÝ mắi cho cịc nhộ trđêng yếu toịn. Thi


tiạp sục ệăng ệéi yếu cẵu cịc thÝ sinh võa giời toịn nhanh võa phèi hĩp lộm viỷc nhãm thẺt


tèt.

n ệỡnh ngộy khai mỰc 8.6.

n ệỡnh mẫi ệéi 6 thÝ sinh.

n ệỡnh tử lỷ huy chđểng Vộng,


BỰc, ậăng lộ 1 : 2 : 3 trến tững sè 60% thÝ sinh. Chử cã sè bội toịn ệở tẽng tõ 6 bội toịn


lến 14 răi 16 bội, thi tiạp sục ệăng ệéi tẽng tõ 6 cẹu lến 12 cẹu vÉn lộm trong 30 phót. Sục


nẳng cựa ệoộn tộu tẽng dẵn. Bẽng qua Long An, Lộo Cai, Cộ Mau, Vỵnh Phóc. Tõ 6 bội


toịn lến tững céng 56 bội toịn cho 4 cuéc thi ệăng thêi 2 cÊp khi tộu tắi ậớk Lớk hềm nay.



BÀI PHÁT BIỂU KHAI MẠC



OLYMPIC TỐN TUỔI THƠ TOÀN QUỐC 2014


CỦA TỔNG BIÊN TẬP TẠP CHÍ




</div>
<span class='text_page_counter'>(10)</span><div class='page_container' data-page=10>

9



Sè toa tộu tõ 15 nay ệở thộnh 34. Nhọng con sè Êy tù nã nãi lến rỪng cuéc thi ệở cã sục


hót, cã sục lan táa. 390 hảc sinh hềm nay ệạn vắi Olympic Toịn Tuữi thể lộ theo ệóng tinh


thẵn Olympic. ậã lộ tù nguyỷn, ệã lộ vđĩt lến chÝnh mừnh, ệuữi kỡp bỰn bÌ vộ nèi vưng tay


bỰn bÌ lộm mét cuéc thi ệẵy chÊt trÝ tuỷ, mét vẽn hãa toịn hảc hđắng tuữi trĨ tắi tđểng lai.


Sù cã mẳt cựa ệỰi diỷn Vô tiĨu hảc, cựa lởnh ệỰo tửnh vộ Sẻ Giịo dơc & ậộo tỰo ậớk Lớk,


cựa cịc thẵy cề giịo, cịc nhộ bịo, cịc từnh nguyỷn viến, cịc phô huynh hềm nay ẻ ệẹy


thÓ hiỷn sù chẽm lo cho nhọng mẵm non toịn hảc. Sù quan tẹm cựa cịc vỡ lởnh ệỰo, cựa


cịc thẵy cề giịo, sù hộo hiỷp cựa nhộ tội trĩ thựy chung ệăng hộnh bÒn bử cỉng Toịn Tuữi


thể 4000 ngộy ệở lộ sù ệờm bờo cho sục sèng cựa cuéc thi, cho thộnh cềng cựa Olympic


Toịn Tuữi thể.

Ê

n tđĩng vỊ chuyạn ệi dội, vđĩt nói bẽng ệÌo hay ệđĩc chiếm ngđìng nhọng


tẵng mẹy, Ên tđĩng vỊ cẵu treo Buền ậền vộ nớng vộng, trêi xanh Tẹy Nguyến chớc sỳ


theo nhiÒu bỰn nhá vộ thộnh kÝ ục khã quến. Nạu sau nộy cịc bỰn nhá ệã trẻ thộnh thẵy


cề, thộnh nhộ khoa hảc thừ mang tiạp ngản lỏa ệã truyÒn cho nhọng lụa tuữi thể sau. Vẹng,


ệạn vắi Toịn Tuữi thể ệÓ ệạn vắi nhọng đắc mể. Cờm ển Bé Giịo dôc vộ ậộo tỰo, UBND


tửnh ậớk Lớk, NXBGD Viỷt Nam, Sẻ Giịo dôc & ậộo tỰo ậớk Lớk vộ cịc tửnh thộnh, Cềng


ty CP VPP Hăng Hộ, Trđêng THCS Tẹn Lĩi, ậội truyÒn hừnh Viỷt Nam vộ cịc nhộ bịo TW


vộ ệỡa phđểng, cịc bỰn từnh nguyỷn viến vộ tÊt cờ mải ngđêi ệở chung lo, chung sục lộm


nến cuéc thi. Huy chđểng khềng quan trảng bỪng ngản lỏa yếu toịn, yếu khoa hảc mộ


Olympic Toịn Tuữi thể ệở thớp lến hển 3000 ngộy qua. Khai mỰc Olympic Toịn Tuữi thể tỰi


ậớk Lớk vắi 34 tửnh thộnh, 29 ệéi THCS, 36 ệéi tiÓu hảc ệở lộ thộnh cềng.



Chóc cịc em hảc sinh lộm bội tèt, vđĩt qua chÝnh mừnh, giộnh giời cao nhÊt cho mừnh, cho


trđêng, cho tửnh, thộnh mừnh ệÓ cuéc thi thộnh cềng hển nọa.



Thay mẳt Ban tữ chục xin ệđĩc chóc sục kháe tÊt cờ mải ngđêi, vộ tuyến bè khai mỰc


Olympic Toịn Tuữi thể toộn quèc 2014 tỰi ậớk Lớk.



</div>
<span class='text_page_counter'>(11)</span><div class='page_container' data-page=11>

10




Thða các vị đại biểu, các vị khách mời cùng toàn thể lãnh đạo các đoàn, các đội,


các giám khảo, giám thị và các tình nguyện viên. Các em thí sinh thân mến.



Vậy là kì thi Olympic Tốn Tuổi thơ toàn quốc 2014 đã kết thúc, sau bao ngày hồi


hộp mong chờ của tất cả chúng ta.



Nẽm nay lộ kừ Olympic lẵn thụ 10.

kừ ệẵu tiến, cã 150 thÝ sinh cựa 15 ệéi tham gia


vộ ệÒ thi găm 5 cẹu tù luẺn cho hảc sinh TiÓu hảc. ậạn kừ Olympic lẵn nộy, ệÒ thi


ệở tẽng lến 56 cẹu cho cờ hai cÊp TiÓu hảc vộ bẺc THCS, vắi 390 thÝ sinh cựa 65


ệéi thuéc 34 tửnh thộnh tham dù. Sù ệữi mắi cựa cịch ra ệÒ thi, cịch tữ chục thi


Olympic Toịn Tuữi thể ệở ngộy cộng thu hót nhiỊu hển sè ệéi tham dù. ậiĨm ệữi


mắi cựa quy chạ trong kừ Olympic lẵn nộy gióp tẽng sè ệéi tham dù cựa mét sè tửnh


thộnh. Nạu nhđ ẻ kừ Olympic trđắc, tửnh chự nhộ ệẽng cai vộ nhọng tửnh thộnh cã


dẹn sè trến 3 triỷu ngđêi ệđĩc quyÒn cỏ 2 ệéi cho cÊp TiÓu hảc vộ 2 ệéi cho bẺc


THCS thừ nẽm nay, cã thếm tửnh ệở tham dù ệự 9 kừ Olympic trđắc còng ệđĩc cỏ 2


ệéi cho mẫi cÊp.



Nẽm nay, thi Tiạp sục ệăng ệéi còng thay ệữi so vắi nẽm trđắc. Thay vừ mẫi thÝ sinh


giời mét bội toịn thừ nay tẽng lến thộnh 2 bội. Nhđ vẺy, ệÒ thi Tiạp sục ệăng ệéi mẫi


cÊp sỳ tẽng thếm 6 bội toịn, gióp phữ ệiĨm réng hển, tẽng tÝnh phẹn loỰi cựa cịc


ệéi.



Kì thi năm nay đã diễn ra an toàn, nghiêm túc. Thay mặt Ban tổ chức, xin chúc


mừng những đội có thí sinh đạt điểm cao nhất phần thi cá nhân là:



+ CÊp tiÓu hảc: Cã 3 hảc sinh ệỰt ệiÓm tuyỷt ệèi 100/100 lộ cịc em: ậoộn Lế Minh


(ậăng Nai), PhỰm Thanh Hời Bừnh (Bừnh Dđểng), NguyÔn Họu Nam (Thanh Hoị).


ậẹy lộ nẽm thụ 2 em PhỰm Thanh Hời Bừnh dù thi vộ lẵn thụ hai ệoỰt huy chđểng


Vộng (Nẽm nay em PhỰm Thanh Hời Bừnh ệang hảc lắp 4).




+ Cấp THCS: Có 2 em học sinh đạt điểm cao nhất 90/100 là các em: Đỗ Hữu Đạt


(Đà Nẵng), Nguyễn Hoàng Hải (Phú Thọ).



Ngay sau đây chúng ta sẽ biết kết quả cụ thể phần thi cá nhân và Tiếp sức đồng


đội. Xin cảm ơn và chúc sức khỏe các vị đại biểu, các thầy cô giáo và các em học


sinh. Xin mời MC.



BÀI PHÁT BIỂU BẾ MẠC



OLYMPIC TỐN TUỔI THƠ TOÀN QUỐC 2014



</div>
<span class='text_page_counter'>(12)</span><div class='page_container' data-page=12>

11



Ngày 5.6.2014, đồn tạp chí Tốn Tuổi thơ đến Bn Ma Thuột, Đắk Lắk.


Chiều ngày 6.6.2014, tại Sở Giáo dục và Đào tạo Đắk Lắk, Ban tổ chức (BTC) Olympic Tốn
Tuổi thơ tồn quốc 2014 gồm các cán bộ Tạp chí, Sở GD - ĐT Đắk Lắk, nhà tài trợ chính Cơng
ty cổ phần Văn phòng phẩm Hồng Hà đã họp để thống nhất những phần việc cuối cùng cho
công tác tổ chức. Sân bay Buôn Ma Thuột tấp nập phụ huynh và học sinh.


Sịng ngộy 7.6.2014, BTC tững duyỷt LÔ khai mỰc vộ LÔ bạ mỰc tỰi Héi trđêng Nhộ vẽn hãa
tửnh ậớk Lớk.


ChiÒu ngộy 7.6.2014, tỰi khịch sỰn Cềng ệoộn Ban Mế, BTC ệở tữ chục ệãn cịc ệoộn tham
dù vộ phịt quộ tẳng cho mẫi ệoộn. Cịc từnh nguyỷn viến ệở hđắng dÉn tẺn từnh vộ gióp cịc
ệoộn. Sau ệã, BTC ệở hảp vắi cịc trđẻng ệoộn, lởnh ệéi phữ biạn quy chạ, ệiỊu lỷ thi vộ nếu
TiĨu hảc, Bé Giịo dơc vộ ậộo tỰo ệở cã nhọng phịt biĨu ệịnh giị tèt cịc hoỰt ệéng cựa TỰp
chÝ vộ phong trộo Olympic Toịn Tuữi thể. Trong buữi gẳp mẳt, cịc ệoộn cịng ệở cã nhọng phịt
biĨu, ệãng gãp ệĨ Olympic TTT ngộy cộng ệđĩc tữ chục tèt hển. Cềng ty cữ phẵn VPP Hăng


Hộ ệở chiếu ệởi cịc trđẻng ệoộn, lởnh ệéi vộ BTC.


Sịng ngộy 8.6.2014, BTC ệở ệi kiÓm tra trđêng thi vộ kiÓm tra héi trđêng chuÈn bỡ cho LƠ khai
mỰc.


ChiỊu ngộy 8.6.2014, LƠ khai mỰc Olympic Toịn Tuữi thể ệở diÔn ra trong khềng khÝ vui tđểi,
hịo hục cựa BTC còng nhđ cịc ệoộn tham dù. Tham dù LƠ khai mỰc, ệỰi diỷn Bé Giịo dơc vộ
ậộo tỰo cã ThS. Ngun ậục Họu, Phã Vơ trđẻng Vơ Giịo dơc TiĨu hảc; TS. TỰ Ngảc TrÝ,
chuyến viến Vơ Giịo dơc TiĨu hảc. VỊ phÝa ệển vỡ ệẽng cai cã bộ Mai Hoan Niế Kdẽm, Phã
ậớk Lớk; ậỰi diỷn cịc ban, ngộnh, ệoộn thÓ cựa tửnh; ậỰi diỷn cịc phưng, ban cựa Sẻ GD - ậT,
cịc Phưng GD - ậT cịc huyỷn, thộnh phè vộ cịc cể sẻ giịo dôc trong tửnh. ậỰi diỷn nhộ tội
Trđểng Quang Luyạn, Chự tỡch Héi ệăng quờn trỡ, Phã Tững Giịm ệèc Cềng ty CP VPP Hăng
Hộ. Mẻ ệẵu LÔ khai mỰc lộ mét sè tiạt môc vẽn nghỷ cựa cịc bỰn thiạu nhi, hảc sinh ậớk Lớk
chộo mõng cịc ệoộn vÒ tham dù Olympic. Tiạp theo, ThS. Vò Kim Thựy, Tững biến tẺp tỰp chÝ
Toịn Tuữi thể, Trđẻng Ban tữ chục phịt biÓu khai mỰc vộ tững kạt 9 nẽm Olympic Toịn Tuữi
thể toộn quèc. Ngay sau ệã, ềng Bỉi Kú Phịt, Tững Giịm ệèc Cềng ty CP VPP Hăng Hộ ệở
phịt biÓu khỬng ệỡnh sù ựng hé cựa Cềng ty vắi hộnh trừnh thi Olympic TTT toộn quèc vộ coi
ệã lộ hoỰt ệéng xở héi bữ Ých. ThS. Vò Kim Thựy ệở lến tẳng quộ cờm ển ệển vỡ tội trĩ. Tiạp
sau lộ gẳp gì bộn trưn trong khuền khữ hoỰt ệéng kử niỷm 10 nẽm Olympic Toịn Tuữi thể. Cề
Trẵn Thỡ Kim Cđểng ệở tữ chục hoỰt ệéng ệè vui rÊt vui nhén lộm cịc bỰn hảc sinh rÊt hẽng
hịi tham gia trờ lêi. TỰi cuéc gẳp gì bộn trưn, ThS. Vò Kim Thựy, ềng Bỉi Kú Phịt, ệỰi diỷn mét
sè tửnh cã phong trộo Olympic TTT phịt triÓn mỰnh; bỰn Lế TuÊn Anh, sinh viến ậỰi hảc NgoỰi
thđểng Hộ Néi, tõng ệoỰt giời BỰc nẽm 2006 vộ giời Vộng nẽm 2010 khi tham dù Olympic TTT;
bỰn TỰ Lế Ngảc Sịng, hảc sinh lắp 7E, trđêng THPT chuyến Hộ Néi - Amsterdam, 2 nẽm liỊn


HÀNH TRÌNH



</div>
<span class='text_page_counter'>(13)</span><div class='page_container' data-page=13>

12



ệỰt giời Thi giời toịn qua thđ (giời BỰc nẽm hảc 2012 - 2013 vộ giời Vộng nẽm hảc 2013 - 2014);


bỰn PhỰm Thanh Hời Bừnh, hảc sinh lắp 4 ệoộn Bừnh Dđểng, tõng ệoỰt giời Vộng Olympic TTT
toộn quèc nẽm 2013 ệở cỉng trao ệữi, thờo luẺn vÒ viỷc hảc vộ dỰy toịn, cịc kinh nghiỷm hảc
tẺp còng nhđ nhọng phđểng phịp ệÓ phịt triÓn khờ nẽng hảc toịn cựa cị nhẹn còng nhđ phong
trộo hảc toịn.


Ngay sau ệã, ệỰi diỷn cựa cịc Sẻ GD - ậT tõng ệẽng cai tữ chục Olympic TTT ệở lến sẹn khÊu
ệÓ nhẺn quộ tẳng cựa BTC: Nam ậỡnh, Quờng Ninh, Hời Phưng, Hộ Néi, Thõa Thiến - Huạ,
Long An, Lộo Cai, Cộ Mau, Vỵnh Phóc, ậớk Lớk. Tiạp theo, ệỰi diỷn cịc tửnh Lộo Cai, Thịi Bừnh,
Thanh Hãa, Vỵnh Phóc, lộ nhọng ệển vỡ tham dù ệẵy ệự Olympic còng lến nhẺn quộ cựa BTC.
Cịc tửnh cã tững sè huy chđểng nhiÒu nhÊt trong 10 nẽm Olympic TTT lộ Hộ Néi, Thanh Hãa,
Vỵnh Phóc, Hời Phưng, Quờng Ninh còng ệđĩc nhẺn quộ cựa BTC. Thay mẳt TỰp chÝ TTT, ThS.
Vò Kim Thựy còng ệở tẳng quộ ệỰi diỷn Sẻ GD - ậT ậớk Lớk, ệển vỡ ệở ệẽng cai Olympic nẽm
nay. Cuèi LÔ khai mỰc lộ hoỰt ệéng diÔu hộnh trến sẹn khÊu cựa cịc ệoộn tham dù. Sẻ GD
-ậT ậớk Lớk ệở chiếu ệởi cịc trđẻng ệoộn, lởnh ệéi vộ BTC.


Ngộy 9.6.2014, cịc thÝ sinh cã mẳt tỰi trđêng thi tõ sắm. Ngay sau LÔ khai mỰc trđêng thi, cịc
thÝ sinh thùc hiỷn bội thi cị nhẹn trong 30 phót. Tiạp sau, cịc từnh nguyỷn viến nhanh chãng
hđắng dÉn cịc thÝ sinh xuèng sẹn trđêng thi ệÓ thi Tiạp sục ệăng ệéi. Sau buữi thi, cịc ệoộn tù
do tham quan. Hẵu hạt cịc ệoộn ệÒu ệạn cẵu treo Buền ậền (Bờn ậền). NhiỊu bỰn ệở ệđĩc
cđìi voi léi si.


Sịng ngộy 10.9.2014, LÔ bạ mỰc Olympic Toịn Tuữi thể toộn quèc 2014 ệở diÔn ra tỰi Héi
trđêng Nhộ vẽn hãa tửnh ậớk Lớk. Mẻ ệẵu buữi lƠ lộ cịc tiạt mơc vẽn nghỷ chộo mõng thộnh
cềng cựa Olympic. Tiạp theo, ThS. Vò Kim Thựy cềng bè kạt quờ vộ phịt biÓu tững kạt Olympic
TTT toộn quèc 2014. Tiạp theo lộ cịc tiạt môc trao thđẻng thi Tiạp sục ệăng ệéi, thi cị nhẹn
cho cịc ệoộn vộ cịc cị nhẹn ệoỰt giời. Sau LÔ bạ mỰc, BTC ệở chiếu ệởi cịc trđẻng ệoộn, lởnh
ệéi, cịc ệỰi biÓu, cịc thẵy cề giịo, cịc từnh nguyỷn viến vộ cịc thÝ sinh dù thi.


PV

THƯ CẢM ƠN




Ban tữ chục Olympic Toịn Tuữi thể toộn quèc lẵn thụ 10 nẽm 2014 xin chẹn thộnh cờm ển sù
gióp ệì hiỷu quờ cựa cịc cể quan trung đểng vộ ệỡa phđểng: Bé GD vộ ậT; UBND tửnh ậớk
Lớk; Sẻ GD - ậT ậớk Lớk; Phưng GD - ậT thộnh phè Buền Ma Thuét; trđêng THCS Tẹn Lĩi,
TP. Buền Ma Thuét; Cịc từnh nguyỷn viến; Nhộ xuÊt bờn Giịo dôc Viỷt Nam; Cềng ty cữ phẵn
Vẽn phưng phÈm Hăng Hộ, nhộ tội trĩ chÝnh; Cịc nhộ tội trĩ: Cềng ty cữ phẵn Dỡch vô XuÊt
bờn Giịo dôc Hộ Néi; Cềng ty cữ phẵn ậộo tỰo - Phịt triĨn Giịo dơc Hộ Néi; Cềng ty cữ phẵn
Bờn ệă - Tranh ờnh Giịo dơc; Cềng ty cữ phẵn ậộo tỰo - Phịt triĨn Giịo dôc ậộ Nơng; Cềng
ty cữ phẵn Sịch - Thiạt bỡ Giịo dơc MiỊn Nam; XÝ nghiỷp Bờn ệă 1, Bé Quèc phưng; Thềng tÊn
xở Viỷt Nam; ậội truyÒn hừnh Viỷt Nam; Bịo Giịo dơc vộ Thêi ệỰi; ậội trun hừnh ậớk Lớk; cịc
cể quan thềng tÊn, bịo chÝ, truyÒn thềng vộ ệẳc biỷt lộ sù tham gia cựa cịc ệoộn ệạn tõ cịc
tửnh thộnh trến cờ nđắc. Xin cờm ển cịc vỡ Trđẻng ệoộn, Lởnh ệéi, cịc giịm khờo, giịm thỡ, cịc
thẵy cề giịo, cịc em hảc sinh vộ cịc vỡ phô huynh. Ban tữ chục xin cờm ển ềng Ngun Xuẹn
Phóc,ựy viến Bé chÝnh trỡ, Phã Thự tđắng ChÝnh phự ệở gỏi lơng hoa chóc mõng Olympic Toịn
Tuữi thể. Xin cờm ển cịc vỡ khịch quý ệở ệạn tham dù vộ hĩp tịc cỉng Ban tữ chục ệÓ Olympic
Toịn Tuữi thể toộn quèc 2014 ệđĩc diÔn ra thộnh cềng tèt ệứp.


</div>
<span class='text_page_counter'>(14)</span><div class='page_container' data-page=14>

Cềng ty Cữ phẵn TruyÒn hừnh Tđểng tịc Viỷt
Nam - VTVlive ệđĩc thộnh lẺp thịng 7.2012,
trùc thuéc Tững Cềng ty TruyÒn hừnh cịp Viỷt
Nam - VTVcab. VTVlive hoỰt ệéng trong lỵnh
vùc nghiến cụu, chun giao cềng nghỷ trun
thềng, trun hừnh; ậẵu tđ phịt triÓn cịc hỰ
tẵng truyÒn thềng ệa phđểng tiỷn; Phịt triÓn
cềng nghiỷp néi dung sè, cể sẻ dọ liỷu truyÒn
thềng ệa phđểng tiỷn; Cung cÊp néi dung vộ
phịt triĨn mỰng lđắi. Vắi viỷc kạ thõa vộ phịt
triĨn ệỊ tội nghiến cụu khoa hảc cÊp nhộ nđắc
“Nghiến cụu, triĨn khai cịc dỡch vơ tđểng tịc
trến nÒn IP”, VTVlive ệđĩc cÊp giÊy chụng nhẺn


Doanh nghiỷp Khoa hảc Cềng nghỷ nẽm 2013
vộ ệđĩc ịp dông cịc chÝnh sịch đu ệởi cựa Nhộ
nđắc ệèi vắi loỰi hừnh doanh nghiỷp nộy. XuÊt
phịt tõ mong muèn mang ệạn cho khịch hộng
trời nghiỷm giời trÝ mắi mĨ cỉng sù chự ệéng
tđểng tịc vộ chia sĨ, VTVlive cung cÊp dỡch vơ
trun hừnh chÊt lđĩng cao dùa trến hỰ tẵng tiếu
chuÈn hãa hộng ệẵu tỰi Viỷt Nam. VTVlive
cung cÊp dỡch vơ trun hừnh vắi môc tiếu
khềng chử cời tiạn vÒ mẳt chÊt lđĩng mộ cưn
hđắng tắi sù phịt triĨn vộ ệữi mắi cịch trun
tời néi dung, tẽng cđêng tÝnh cị thÓ hãa, tđểng
tịc vộ chia sĨ tắi ngđêi dỉng nhỪm xẹy dùng
hừnh ờnh truyÒn thềng mắi - TruyÒn thềng ệa
nỊn tờng, ệa phđểng tiỷn vắi sù héi tơ cựa viƠn
thềng, trun hừnh vộ internet. Trong thịng
6.2014, tỰp chÝ Toịn Tuữi thể vộ VTVlive ệở gẳp
gì, trao ệữi vộ triĨn khai hĩp tịc ệÓ tữ chục cịc
cuéc thi, cịc chđểng trừnh dỰy toịn tđểng tịc
trến truyÒn hừnh vộ mịy vi tÝnh.


Song song vắi quị trừnh phịt triÒn cựa Khoa
hảc vộ Cềng nghỷ, cịc hừnh thục dỰy hảc cho
trĨ em ngộy cộng ệa dỰng vộ phong phó.
POMATH lộ chđểng trừnh dỰy tđ duy cho trĨ
thềng qua dỰy hảc mền toịn. Chđểng trừnh ệở
ệđĩc Viỷn Nghiến cụu vộ ChuyÓn giao Cềng
nghỷ Giịo dơc kiĨm ệỡnh vộ thềng qua.
“POMATH” viạt tớt tiạng Anh cựa Improving



Mathematical thinking with Personal Oriented
program for children - Chđểng trừnh phịt triÓn tđ
duy thềng qua dỰy hảc mền toịn theo ệỡnh
hđắng cị nhẹn dộnh cho trĨ em. ậã lộ kạt quờ
nghiến cụu vộ Êp ự tõ nẽm 2002 cựa TS. Chu
CÈm Thể vộ cịc céng sù thuéc khoa Toịn tin
-Trđêng ậHSP Hộ Néi. Vắi kinh nghiỷm giờng
dỰy lẹu nẽm, cề hiÓu rỪng trĨ em khềng hÒ ghĐt
toịn, chÝnh cịch tiạp cẺn toịn hảc mộ chóng ta
ệang thùc hiỷn mang ệạn nẫi sĩ hởi lắn hển vỊ
toịn cho trĨ. Vắi khịt vảng gióp trĨ thoời mịi
khi hảc toịn, biạn toịn trẻ thộnh cềng cô hẫ trĩ
cho sù thộnh cềng cựa cịc em chụ khềng phời
biạn cịc em thộnh nề lỷ cựa toịn hảc, cề ệở
dộnh toộn bé tẹm huyạt cựa mừnh ệÓ theo ệuữi
khịt vảng ệã. Chđểng trừnh ệở ệđĩc ệèi chiạu
vắi cịc chđểng trừnh dỰy hảc tiến tiạn cựa
Mủ, NhẺt, ậục, Singapore,... nhỪm ệịp ụng
ệđĩc yếu cẵu vÒ chuÈn Quèc gia vộ Quèc tạ.
Tõ TTT1 sè 167 (thịng 9 nẽm 2014) tỰp chÝ
Toịn Tuữi thể sỳ ệẽng cịc bội giờng theo
phđểng phịp dỰy hảc mắi cựa trung tẹm toịn
POmath.


SmartEbook.com Viỷt Nam lộ mét cềng ty
100% vèn nđắc ngoội cã chự sẻ họu lộ Cềng ty
TNHH SmartEbook.com NhẺt Bờn, chuyến
cung cÊp giời phịp nÒn tờng sịch ệiỷn tỏ
Ebook. Hiỷn tỰi SmartEbook cã rÊt nhiÒu kếnh
phẹn phèi tỰi cịc quèc gia khịc nhau. Thịng


10.2012, SmartEbook ệở niếm yạt trến thỡ
trđêng chụng khoịn NhẺt Bờn Jasdaq. Khềng
ngõng nẹng cao chÊt lđĩng vộ phịt triÓn liến
tôc, môc tiếu cựa SmartEbook lộ trẻ thộnh nhộ
cung cÊp hộng ệẵu trong lỵnh vùc sịch ệiỷn tỏ
trến thạ giắi. Tõ thịng 3.2014, tỰp chÝ Toịn Tuữi
thể ệở hĩp tịc vắi SmartEbook.com Viỷt Nam
ệÓ ệđa cịc sè cựa tỰp chÝ lến mỰng
internet vộ ệiỷn thoỰi di ệéng. Cịc bỰn cã thÓ
ệảc tỰp chÝ Toịn Tuữi thể Online ẻ cịc trang
web:oẳc .
PV

13



</div>
<span class='text_page_counter'>(15)</span><div class='page_container' data-page=15>

14


Cẹu 1. Từm sè tù nhiến n cã hai chọ sè sao cho
2n 1 vộ 3n 1 ệÒu lộ cc số chính phng.


Câu 2. Tìm các số nguyên x và y tháa m·n
6xy 4x 9y 7 0.


Cẹu 3. Từm giị trỡ nhá nhÊt cựa biÓu thục
A x3 y3 xy, trong ệã x, y lộ cịc sè dđểng
tháa mởn ệiÒu kiỷn x y 1.


Câu 4. Tìm hai số tận cùng bên phải cđa sè
10072014.


Cẹu 5.Hừnh bến biĨu diƠn mét bộn trưn vắi 8 ghạ
ệịnh sè thụ tù tõ 1 ệạn 8. Cã 2 sinh viến ệđĩc


xạp ngăi ệèi diỷn nhau vộ ệÓ trèng cịc ghạ cưn
lỰi. Hái tÊt cờ cã bao nhiếu cịch xạp?


Cẹu 6.Giờ sỏ x<sub>1</sub>, x<sub>2</sub>, x<sub>3</sub>lộ ba nghiỷm cựa phđểng
trừnh x3 x 1 0. TÝnh giị trỡ cựa biểu thc


.


Câu 7.Cho hình vuông ABCD. Gọi K, L, M, N là
các điểm trên cạnh BC sao cho


BK KL LM MN NC.


Trên cạnh AD lấy điểm E sao cho AE BK. TÝnh
tỉng c¸c gãc


Câu 8. Tìm các số nguyên x, y thỏa mãn
sao cho tích xy đạt giá trị ln
nht.


Câu 9. Cho tam giác ABC có Tia
phân giác trong của góc A cắt cạnh BC ở D. Tìm
số đo góc


Câu 10. Cho tứ giác ABCD thỏa mÃn điều kiện
Tính số đo góc B.


A B C D.
6 5 4 3



ADC.


o


B C 20 .


2
2


2


1 y


2x 4


4
x


AKE ALE AME ANE ACE.


3


1 2


1 2 3


1 x
1 x 1 x


T



1 x 1 x 1 x


OLYMPIC TOÁN TUỔI THƠ TOÀN QUỐC 2014 - THCS



Thêi gian lµm bµi: 30 phót


ậỊ thi găm 2 trang. Hảc sinh khềng ệđĩc sỏ dông mịy tÝnh.
Tõ cẹu 1 ệạn cẹu 15 chử viạt p số.


</div>
<span class='text_page_counter'>(16)</span><div class='page_container' data-page=16>

15


Câu 11.Cho hình vuông ABCD và E là điểm trên
cạnh AD sao cho S<sub>ABCD</sub> 144 cm2 vµ


Tìm độ dài đoạn AE.


Cẹu 12. Trong hừnh vỳ bến, mẫi cỰnh cựa hừnh
vuềng ệÒu ệđĩc chia thộnh 3 ệoỰn bỪng nhau.
TÝnh tử sè giọa diỷn tÝch phẵn ệđĩc tề ệẺm vắi
diỷn tÝch hừnh vuềng lắn.


Cẹu 13. Hừnh vỳ bến ệđĩc tỰo bẻi mét lơc giịc
ệỊu A, mét hừnh vuềng B vộ mét tam giịc ệÒu C.
Từm sè ệo gãc x.


Cẹu 14.Trong sể ệă bến, cịc sè ệđĩc xạp theo
mét quy luẺt. Từm sè x phỉ hĩp vắi quy luẺt ệã.


Cẹu 15. Cho tam giịc ABC cẹn tỰi C. KĨ trung
tuyạn CM vộ ệđêng phẹn giịc trong AD. TÝnh sè


ệo gãc biạt rỪng AD 2CM.


Cẹu 16 (Tù luẺn).Mét bờng hừnh chọ nhẺt ABCD
cã AB 2013 cm, AD 2014 cm. Mét con kiạn
xuÊt phịt tõ ệiÓm A, ệi theo ệđêng chĐo cựa mẫi
ề vuềng (hừnh bến). Hái rỪng sau khi ệi qua tÊt cờ
cịc ề vuềng mộ mẫi ề vuềng chử ệi qua mét
ệđêng chĐo thừ con kiạn cã thÓ ệạn ệiĨm nộo
trong ba ệiĨm B, C, D.


Chó ý: C¸n bộ coi thi không giải thích gì thêm.


B


ABE 1 ABCD


</div>
<span class='text_page_counter'>(17)</span><div class='page_container' data-page=17>

16



Đề bài: Biết rằng T 1,9565656... (viết liên tơc hai ch÷ sè 56).
a) TÝnh 1000T 10T.


b) Tõ ệã biĨu diƠn T dđắi dỰng mét phẹn sè .


Đề bài: Cho ABCD là hình vng cạnh bằng 1. Biết EFC là tam giác đều và chiều dài cạnh AE là a.
a) Tính diện tích tam giác EBC theo a.


b) Tìm giá trị của biểu thức .


Đề bài:



a) Rót gän (m 2)2 m(m 4).


b)áp dụng kết quả cõu a) tớnh 20122 2014.2010.
2


a 1


a



a (a,b )


b



Đáp số: ...


Điểm số: ...


Giám khảo


(Kí và ghi rõ họ tên)


Đáp số: ...


Điểm số: ...


Giám khảo


(Kí và ghi rõ họ tên)


Đáp số: ...


Điểm số: ...



Giám khảo


(Kí và ghi rõ họ tên)


OLYMPIC TON TUỔI THƠ TOÀN QUỐC 2014 - THCS



</div>
<span class='text_page_counter'>(18)</span><div class='page_container' data-page=18>

17


§Ị bài:


a) Biết rằng 2014a 8,06 2005,94 8,062. Tìm giá trị của a.
b) Biết rằng 81x2 108xy 36y2 0. Tìm giá trị của .


ề bi: Ba dng u cựa mét dởy ệđĩc cho nhđ sau
12 02 1 0 1
22 12 2 1 3
32 22 3 2 5
a) Hởy viạt dưng thụ m.


b) Mét dưng ệđĩc viạt ra lộ p2 q2 p q r. Từm p nạu r 191.


Đề bài: Trong hình bên cho DE // CA, FE // GA vµ M lµ trung ®iĨm cđa DE. Cho BE 12 cm,
BG 6 cm, DE 10 cm và CA 15 cm.


a) Tìm chiều dài EA.
b) Tìm chiều dài GD.


x


6y




Đáp số: ...


Điểm số: ...


Giám khảo


(Kí và ghi rõ họ tên)


Đáp số: ...


Điểm số: ...


Giám khảo


(Kí và ghi rõ họ tên)


Đáp số: ...


Điểm số: ...


Giám khảo


</div>
<span class='text_page_counter'>(19)</span><div class='page_container' data-page=19>

18



STT Số báo danh Họ và tên Tỉnh, Thành phố Giải


1 ĐN09 Đỗ Hữu Đạt Đà Nẵng Vàng


2 N08 Nguyễn Tờng Duy Bo Nng Vng



3 ĐN12 Phạm Tuấn Kiệt Đà Nẵng Vàng


4 ĐL10A Lê Ngọc Cảnh Đắk Lắk Vàng


5 BG08 Vng nh n Bc Giang Vng


6 GL08 Huỳnh Xuân Bách Gia Lai Vµng


7 HNa08 PhỰm Thanh Phđểng Hộ Nam Vộng


8 HNa07 Phạm Quỳnh Anh Hà Nam Vàng


9 HP08 Trịnh Khánh Hiệp Hải Phòng Vàng


10 LCa11A Nguyễn Tiến Dũng Lào Cai Vàng


11 LS09 Bùi Minh Quốc Lạng Sơn Vàng


12 NB12 Trần Quang Thiện Ninh Bình Vàng


13 PT07 Nguyễn Hoàng Hải Phú Thọ Vàng


14 PT10 Kiều Tiến Đạt Phú Thọ Vàng


15 PY08 Huỳnh Trung Nghĩa Phú Yên Vàng


16 TH07B Trng Hi Long Thanh Hãa Vộng


17 TH12B Vị §øc ViƯt Anh Thanh Hãa Vàng



18 TH08B Phan Trung Hiếu Thanh Hóa Vàng


19 VP11A Văn Thanh Tùng Vĩnh Phúc Vàng


20 ĐN10 Hồ Nh Hoàng Đà Nẵng Bạc


21 ĐN07 Đặng Minh Anh Đà Nẵng Bạc


22 ĐL12A Nguyễn Đăng Khoa Đắk Lắk Bạc


23 ĐL08A Lê Đỗ Thanh Bình Đắk Lắk Bạc


24 ĐL07A Đặng Tuấn Anh Đắk Lắk Bạc


25 L08B Võ Quang Trờng k Lk Bc


26 ĐL07B Nguyễn Thị Thùy Nhung Đắk Lắk Bạc


27 BD10 Phm Ngc Trinh Bnh Dng Bc


28 BG11 Nguyễn Thế Chính Bắc Giang Bạc


29 BG09 Đinh Công Duy Bắc Giang Bạc


30 HNa10 Nguyễn Bi Nam Trờng H Nam Bc


31 HNa12 Lê Ngọc Trung Hà Nam Bạc


32 HNa09 Nguyễn Thị Huế Hà Nam Bạc



33 HP10 Nguyễn Hải Nam Hải Phòng Bạc


34 LS12 Nghiêm Việt Thắng Lạng Sơn Bạc


35 LS08 Vũ Thu Thảo Lạng Sơn Bạc


36 NB08 Phạm Văn Đức Ninh Bình Bạc


37 NB09 Phạm Tuấn Hiên Ninh Bình Bạc


38 NB11 Phạm Ngọc Quý Ninh Bình Bạc


39 PT08 Lê Hùng Phú Thọ Bạc


40 PT11 Phạm Duy Khánh Phú Thọ Bạc


41 PT09 Hồ Quang Huy Phú Thọ Bạc


42 ST07 Lê Thanh Triều Sóc Trăng Bạc


43 ST12 Trần Ngọc Vinh Sóc Trăng Bạc


44 TNi12 Võ Minh Quân Tây Ninh Bạc


45 TH12A Lê Đức Tùng Thanh Hóa Bạc


</div>
<span class='text_page_counter'>(20)</span><div class='page_container' data-page=20>

19



STT Số báo danh Họ và tên Tỉnh, Thành phố Giải



46 TH10A Trần Thế Lâm Thanh Hóa Bạc


47 TH11A Ngun ThÕ Phïng Thanh Hãa B¹c


48 TH08A Ngun Bïi NhËt Anh Thanh Hãa B¹c


49 TH09A Ngun Ngäc Hun Thanh Hãa Bạc


50 TH09B Đỗ Nam Thanh Hóa Bạc


51 TTH08 Dng Vn i Thừa Thin - Hu Bc


52 VL07 Ngô Hoàng Anh VÜnh Long B¹c


53 VP10A Ngun Xuẹn Dđểng Vỵnh Phóc BỰc


54 VP12A Nguyễn Hồng Anh Vĩnh Phúc Bạc


55 VP07B Hoàng Văn HiÕu VÜnh Phóc B¹c


56 VP10B Lế Tđêng Khanh Vỵnh Phóc Bc


57 VP12B Nguyễn Duy Ngọc Vĩnh Phúc Bạc


58 ĐN11 Phạm Nguyễn Quốc Huy Đà Nẵng Đồng


59 ĐL10B Đỗ Thị Mỹ Hằng Đắk Lắk Đồng


60 ĐL12B Lê Hoàng Phúc Đắk Lắk Đồng



61 ĐL11B Trịnh Thị Thảo Đắk Lắk Đồng


62 BĐ12 Lê Bá Thành Bình Định Đồng


63 BĐ10 Nguyễn Trần Khôi Nguyên Bình Định Đồng


64 BD07 Nguyễn Phi Thn Bnh Dng ng


65 BD08 Ngun Trẵn Tiạn Dịng Bừnh Dđểng ậăng


66 BV08 Ph¹m Duy Hải Bà Rịa - Vũng Tàu Đồng


67 BV11 Nguyễn Hoàng Bảo Bà Rịa - Vũng Tàu Đồng


68 BV07 Nguyễn Tiến Dũng Bà Rịa - Vũng Tàu Đồng


69 BG12 Nguyễn Minh Đức Bắc Giang Đồng


70 GL07 Nguyễn Ngọc Anh Khoa Gia Lai Đồng


71 GL12 Nguyễn Hữu Hoàng Gia Lai Đồng


72 HNa11 PhỰm Phđểng Thờo Hộ Nam ậăng


73 HP12 Vò Cđêng Thnh Hi Phng ng


74 HB09 Bùi Thị Bích Ngọc Hòa Bình Đồng


75 HB12 Nguyễn Quang Trung Hòa Bình Đồng



76 HB07 An Phđểng ậềng Hưa Bừnh ậăng


77 LCa07A Ngun Minh HiƯu Lào Cai Đồng


78 LCa08A Bùi Diệu Linh Lào Cai Đồng


79 LCa09A Trần Khánh Linh Lào Cai Đồng


80 LCa12A Trần Ngọc Minh Lào Cai Đồng


81 LCa08B Lê Tuấn Khang Lào Cai Đồng


82 LCa11B Bùi Đức Khải Lào Cai Đồng


83 LCa10B Nguyễn Quang Tùng Lào Cai Đồng


84 LCa07B Hng Giang Lo Cai ậăng


85 LS11 Trđểng Thóy Quúnh LỰng Sển ậăng


86 LA12 Nguyễn Cao Trí Long An Đồng


87 LA07 Đoàn Linh Huy Long An Đồng


88 NB07 Đinh Quý Đức Ninh Bình Đồng


89 PT12 Lê Bá Hoàng Phú Thọ Đồng


90 PY11 Nguyễn Tạ Hữu Thuyên Phú Yên Đồng



91 PY07 Phạm Lê Ngọc Huyền Phú Yên Đồng


92 PY10 Nguyễn Lê Thảo Vân Phú Yên Đồng


</div>
<span class='text_page_counter'>(21)</span><div class='page_container' data-page=21>

20



STT Số báo danh Họ và tên Tỉnh, Thành phố Giải


94 QB10 Nguyễn Thanh Huyền Quảng Bình Đồng


95 QB08 Nguyễn Tuấn Đạt Quảng Bình Đồng


96 QB11 Nguyễn Quốc Khánh Quảng Bình Đồng


97 SL11 Lê Thu Hà Sơn La Đồng


98 SL07 Nguyễn Yến Khanh Sơn La Đồng


99 ST08 Nguyễn Văn Hoài Sóc Trăng Đồng


100 TNi07 Lê Hiền Khải Tây Ninh Đồng


101 TNi10 Trần Lê Đăng Khoa Tây Ninh Đồng


102 TNi08 Tạ Nguyên Bảo Tây Ninh Đồng


103 TH07A Lê Tuấn Anh Thanh Hóa Đồng


104 TH11B Hà Minh Tùng Thanh Hóa Đồng



105 TTH09 Nguyễn Văn Đông Thừa Thiên - Huế §ång


106 TTH12 Phan Thanh Trđêng Thõa Thiến - Huạ ậăng


107 SG07 Trẵn Thỡ Lan Hđểng TP. Hă ChÝ Minh ậăng


108 SG11 Huúnh Ngäc Minh TP. Hå ChÝ Minh §ång


109 SG09 Trần Trí Thiện TP. Hồ Chí Minh Đồng


110 VL10 Châu Minh Khánh Vĩnh Long Đồng


111 VL09 Ngô Minh Hiền Vĩnh Long Đồng


112 VP08A Hoàng Anh Thái Vĩnh Phúc Đồng


113 VP07A Trung Phng Vnh Phúc ng


114 VP11B Lu Văn Nam Vĩnh Phúc Đồng


115 VP08B Phan Hữu Thi Hoàn Vĩnh Phúc Đồng


116 ĐL11A Mai Đình Trọng Đạt Đắk Lắk Khuyến khích


117 ĐL09A Lê Quang Long Đắk Lắk Khuyến khích


118 ĐL09B Phạm Văn Tín Đắk Lắk Khuyến khích


119 BĐ08 Nguyễn Việt Long Bình Định Khuyến khích



120 B11 Trng Th Ty Thi Bnh ậỡnh Khuyạn khÝch


121 Bậ07 Trđểng Thỡ Thanh Hộ Bừnh ậỡnh Khuyn khích


122 BĐ09 Nguyễn Văn Thiện Bình Định Khuyến khích


123 BD11 Ngun ậinh Vđểng Dịng Bừnh Dđểng Khuyạn khÝch


124 BD09 NguyÔn Thộnh Nhẹn Bừnh Dđểng Khuyạn khÝch


125 BD12 Lế Trđểng Dng Bnh Dng Khuyn khích


126 BV10 Phạm Thanh Danh Bà Rịa - Vũng Tàu Khuyến khích


127 BV09 Phan Nguyễn Vinh Bà Rịa - Vũng Tàu Khuyến khích


128 BV12 Trng Anh Tn Bộ Rỡa - Vịng Tộu Khuyạn khÝch


129 BG07 Hµ Tn Anh B¾c Giang Khun khÝch


130 BG10 Ngun Khđểng Duy Bớc Giang Khuyạn khÝch


131 GL09 Phïng Anh Huy Gia Lai KhuyÕn khÝch


132 GL11 Lª Quang Huy Gia Lai KhuyÕn khÝch


133 GL10 Bùi Tiến Hoàng Gia Lai Khuyến khích


134 HP07 Phạm Thị Hải Hải Phòng Khuyến khích



135 HP11 Vũ Thành Lập Hải Phòng Khuyến khích


136 HB10 Nguyễn Bá Thiên Phong Hòa Bình Khuyến khích


137 HB11 Nguyễn Đức Thành Hòa Bình Khuyến khích


138 HB08 Văn Đức Hoàng Hòa Bình Khuyến khích


139 LCa10A Nguyễn Đức Mạnh Lào Cai Khuyến khích


140 LCa09B H Vng Linh Lộo Cai Khuyạn khÝch


</div>
<span class='text_page_counter'>(22)</span><div class='page_container' data-page=22>

21



Mẫi cẹu ệóng ệđĩc 1 ệiĨm. Mẫi bội ệóng ệđĩc
2 ệiĨm


Bµi 1. a) 1937. b) .


Bµi 2. a) . b) .


Bµi 3. a) 4. b) 4.


Bµi 4. a) 8,06. b) .


Bµi 5. a) m2 (m 1)2 m (m 1) 2m 1.
b) 96.


Bµi 6. a) 6 cm. b) 2 cm.



1
9
1
2
1 a
2
1937
T
990


Từ câu 1 đến câu 15 mỗi câu 5 điểm (thiếu đơn
vị vẫn cho điểm tối đa)


C©u 1.n 40. C©u 2.x 1; y 1.


C©u 3. khi


C©u 4.49. C©u 5.8.


C©u 6.T 7. C©u 7.45o.


C©u 8.(x, y) {(1; 2); ( 1; 2)}.


C©u 9. C©u 10.
C©u 11.8 cm. C©u 12.
C©u 13.45o. C©u 14.x 2.


C©u 15.


Cẹu 16.LẺp hỷ trơc tảa ệé vắi trôc hoộnh AB, trôc


tung lộ AD. Khi ệã A(0; 0), B(2013; 0), C(2013; 2014),
D(0; 2014). Con kiạn ệụng ẻ ệửnh A, khi di chuyÓn
theo ệđêng chĐo cựa mét ề vuềng thừ hoộnh ệé
tẽng (hoẳc giờm) ệi mét ệển vỡ vộ tung ệé tẽng
(hoẳc giờm) ệi mét ệển vỡ. Do ệã tảa ệé cựa con
kiạn luền cã dỰng (lĨ; lĨ) hoẳc (chơn; chơn). VẺy
cuèi cỉng con kiạn sỳ ệạn ệửnh D.


-ýtđẻng vỊ tảa ệé: 5 ệiĨm.


- Gán tọa độ cho các đỉnh A, B, C, D: 5 điểm.
- Tăng giảm tọa độ: 5 điểm.


- Phịt hiỷn tÝnh chơn lĨ: 5 ệiÓm.
- Kạt luẺn ệđĩc ệửnh D: 5 ệiÓm.


Chú ý:Làm cách khác đúng vẫn cho điểm tối đa.
Tổng điểm là 100. Không cho điểm phân số.


o
B 36 .


1.
9


o
B 100 .
o


ADC 100 .



1
x y .


2
1


minA
2


OLYMPIC TỐN TUỔI THƠ TOÀN QUỐC 2014 - THCS



KẾT QUẢ OLYMPIC TỐN TUỔI THƠ TOÀN QUỐC 2014 - THCS


GIẢI TIP SC NG I



Đội Giải


Thanh Hóa B, Lào Cai B, Phú Yên Vàng


Sơn La, Thanh Hóa A, Vĩnh Phúc A, Đắk Lắk B, Lạng Sơn Bạc
Ninh Bình, Vĩnh Long, Đà Nẵng, Phú Thọ, TP. Hồ Chí Minh, Bà Rịa - Vũng Tàu, Hải Phòng, Bình Định Đồng


STT Số báo danh Họ và tên Tỉnh, Thành phố Giải


142 LS07 Mẫn Đào Sơn Tùng Lạng Sơn Khuyến khích


143 NB10 Nguyễn Thị Ngọc Huyền Ninh Bình Khuyến khích


144 PY09 Lê Thị Hồng Huyên Phú Yên Khuyến khích



145 QB12 Thái Trần Huyền Trang Quảng Bình Khuyến khích


146 SL10 Lê Thị Thu Huyền Sơn La Khuyến khích


147 SL09 Trần Thị Thu Thảo Sơn La Khuyến khích


148 ST09 Nguyễn Khánh Vĩnh Sóc Trăng Khuyến khích


149 TH10B Lê Ngọc Đình Thanh Hóa Khuyến khích


150 TTH10 Nguyễn Phúc Anh Minh Thừa Thiên - Huế Khuyến khích


151 VL12 Phạm Hoài Phú Thịnh Vĩnh Long Khuyến khích


152 VL08 Trng Ng Minh Bo Vnh Long Khuyn khích


</div>
<span class='text_page_counter'>(23)</span><div class='page_container' data-page=23>

22


Câu 1.Đặt 2n 1 a2, 3n 1 b2(a, b )
Ta có a là số lẻ nên 2n a2 1 (a 1)(a 1) 8.
Suy ra n 4.


Do 3n 1 lộ sè chÝnh phđểng lĨ nến 3n 1 chia
cho 8 dđ 1.


Suy ra n 8.


Ta lại có a2 b2 5n 2 chia cho 5 dð 2 nên
a2và b2đều chia cho 5 dð 1.


Do đó n 5. Suy ra n 40.



Do ệã n {40; 80}. Thỏ lỰi ta ệđĩc n 40.


Nhận xét: Bài toán này khá quen thuộc, có
49,43% số thí sinh dự thi có kết quả đúng.


C©u 2.Ta cã 6xy 4x 9y 7 0
2x(3y 2) 3(3y 2) 1


(2x 3)(3y 2) 1.
Tõ ệã ta ệđĩc x 1, y 1.


Nhận xét: Có 49,43% số thí sinh dự thi có kết
quả đúng của bi toỏn ny.


Câu 3.Vì x y 1 nên y 1 x
Ta cã A x3 (1 x)3 x(1 x)
x3 1 3x 3x2 x3 x x2
2x2 2x 1 2(x )2
VËy


Nhận xét: Có 50% số thí sinh dự thi có kết quả
đúng bài tốn này.


C©u 4. Ta cã 10072014 72014 (mod 100).
Mặt khác 72014 (74)503.72 2401503.49


.49


Nhn xột:Cú 47,7% s thớ sinh dự thi có kết quả


đúng bài tốn này.


Cẹu 5. Cã 8 cịch xạp ngđêi thụ nhÊt ngăi vộo
mét trong tịm chiạc ghạ ệở cho. Trong mẫi cịch
xạp ngđêi thụ nhÊt thừ cã mét cịch xạp ngđêi thụ
2 ngăi vộo ghạ ệèi diỷn vắi ngđêi thụ nhÊt. VẺy
cã 8 cịch xạp.


NhËn xÐt: Cã 47,13% sè thÝ sinh dù thi cã kÕt


quả đúng. Nhiều em đã có kết quả là 16.


Cẹu 6.Cịch 1.Ta cã x<sub>1</sub>, x<sub>2</sub>, x<sub>3</sub>lộ cịc nghiỷm cựa
phđểng trừnh x3 x 1 0 nến


x3 x 1 (x x<sub>1</sub>)( x x<sub>2</sub>)( x x<sub>3</sub>)


x3 (x<sub>1</sub> x<sub>2</sub> x<sub>3</sub>)x2 (x<sub>1</sub>x<sub>2</sub> x<sub>2</sub>x<sub>3</sub> x<sub>1</sub>x<sub>3</sub>)x
x<sub>1</sub>x<sub>2</sub>x<sub>3</sub>.


Do đó x<sub>1</sub> x<sub>2</sub> x<sub>3</sub> 0; x<sub>1</sub>x<sub>2</sub> x<sub>2</sub>x<sub>3</sub> x<sub>1</sub>x<sub>3</sub> 1;
x<sub>1</sub>x<sub>2</sub>x<sub>3</sub> 1.


Ta l¹i cã


Tđểng tù


Do đó


Cách 2.Đặt t x 1 thì x t 1. Khi đó 1 x t


và 1 x 2 t.


Ta cã phđểng trừnh (t 1)3 (t 1) 1 0
t3 3t2 3t 1 t 1 1 0


t3 3t2 2t 1 0 (1)


1 2 2 3 3 1
1 2 3


1 2 3
x x x x x x
9 2(x x x ) 2


x x x
1


9 2.0 2 7.
1


3
1 2


1 2 3


1 x
1 x 1 x


T



1 x 1 x 1 x
3


3


3 3


1 x <sub>(3 2x</sub> 2 <sub>).</sub>


1 x x


2


2


2 2


1 x <sub>(3 2x</sub> 2<sub>),</sub>


1 x x


3 2


1 1 1 1 1


2
1 1 1 1


1 1



3 2 2


1 1 1 1 1


1 1


1
1


x 1 x (1 x )(1 x x )
1 x (1 x )(1 x x )
1 x x


x 2x 2x 1 2x 3x 2


x x


2
(3 2x ).


x
3
1 1
x x 1 0


...49.
...01


1 1



MinA x y .


2 2


1 1.
2 2
1


2


LTS. Có 174 thí sinh cấp THCS dự thi. Sau đây là lời giải và
nhận xét bài làm của các thí sinh. Điểm của các thí sinh năm
nay trải rộng từ 0 điểm đến 90 điểm. Lời giải chỉ dùng để


tham kh¶o.


O

LYMPIC TỐN TUỔI THƠ 2014



THCS VÀ NHẬN XÉT



HƯỚNG DẪN



</div>
<span class='text_page_counter'>(24)</span><div class='page_container' data-page=24>

23



Giờ sỏ phđểng trừnh (1) cã 3 nghiỷm t<sub>1</sub>, t<sub>2</sub>, t<sub>3</sub> thừ
t3 3t2 2t 1 (t t<sub>1</sub>)( t t<sub>2</sub>)( t t<sub>3</sub>) t3 (t<sub>1</sub>


t<sub>2</sub> t<sub>3</sub>)t2 (t<sub>1</sub>t<sub>2</sub> t<sub>2</sub>t<sub>3</sub> t<sub>1</sub>t<sub>3</sub>)t t<sub>1</sub>t<sub>2</sub>t<sub>3</sub>.


Do đó t<sub>1</sub> t<sub>2</sub> t<sub>3</sub> 3; t<sub>1</sub>t<sub>2</sub> t<sub>2</sub>t<sub>3</sub> t<sub>1</sub>t<sub>3</sub> 2; t<sub>1</sub>t<sub>2</sub>t<sub>3</sub>


1. Suy ra


Nhận xét: Bài tốn này khá khó, địi hỏi các thí
sinh phải có kĩ thuật biến đổi, rất tiếc khơng có thí
sinh nào có kết quả đúng.


Câu 7. Vì các tứ giác EABK, EAKL, EALM,
EAMN và EANC là các hình bình hành nên có
các cặp cạnh đối song song với nhau.


Do đó


Nhận xét: Bài tốn này khá hay, có 42,53% số
thí sinh dự thi có kết quả đúng.


C©u 8.Ta cã


DÊu b»ng x¶y ra khi


Suy ra (x, y) {(1; 2); ( 1; 1)}.


Nhận xét:Bài tốn này có cách biến đổi rất hay,
có 47,13% số thí sinh dự thi có kết quả đúng.


C©u 9.


Ta cã


Mặt khác
Do đó



NhËn xÐt: Cã 61,49% sè thÝ sinh dù thi cã kết


qu ỳng.


Câu 10.áp dụng tính chất dÃy tỉ số bằng nhau ta


Do đó


Nhận xét: Có 72,99% số thí sinh dự thi có kết
quả đúng.


C©u 11. Ta cã S<sub>ABCD</sub> AD2 144 cm2 nên
AD 12 cm.


Mặt khác nªn


Nhận xét:Bài tốn này dễ, có 81,03% số thí sinh
dự thi có kết quả đúng. Tức là vẫn có 18,97% các
thí sinh cho kết quả sai.


C©u 12.


Ta cã tam giác MNP vuông cân tại P nên


Do ó t số giọa diỷn tÝch phẵn ệđĩc tề ệẺm vắi
diỷn tÝch hừnh vuềng lắn lộ


Nhận xét: Giải bài toán này cần nhớ cơng thức


tính diện tích tam giác vng cân theo cạnh huyền,
có 39,08% số thí sinh dự thi có kết quả đúng.


C©u 13.


Lục giác đều có số đo một góc là 120o.
Hình vng có số đo một góc là 90o.
Tam giác đều có số đo một góc là 60o.
Do đó PMN 360o 120o 90o 60o 90 .o


4 <sub>1.</sub>
36 9
2


2


2


MNP MN 1 AB 1 1 ABCD


S AB S .


4 4 3 36 36


1<sub>AB.AE</sub> 1<sub>AB.AD</sub> <sub>AE</sub> 2<sub>AD</sub> 2<sub>.12 8cm.</sub>


2 3 3 3


ABE 1 ABCD
S S



3
o o
B 5.20 100 .


o <sub>o</sub>
A B C D A B C D 360 <sub>20 .</sub>
6 5 4 3 6 5 4 3 18


o o o


ADC (180 20 ) : 2 100 .
o


ADC ADB 180 .


o


ADC ADB (B BAD) (C CAD) B C 20 .


2 2


1 y


x x 0.


x 2
2
2 2
2


2 2
1 y


2 xy x 2 x xy
4
x


1 y


x x 0 xy 2.


x 2
2
2
2
1 y
2x 4
4
x
o
KAB LAK MAL NAM CAN 45 .


AKE ALE AME ANE ACE


3 3


1 2 1 2


1 2 3 1 2 3



1 2 2 3 3 1


1 2 3 1 2 3


1 x 2 t
1 x 1 x 2 t 2 t


T


1 x 1 x 1 x t t t
t t t t t t
1 1 1


2( ) 3 2 3


t t t t t t
2


</div>
<span class='text_page_counter'>(25)</span><div class='page_container' data-page=25>

24



Suy ra tam giác PMN vuông cân t¹i M.
VËy x 45o.


Nhận xét: Có 75,86% số thí sinh dự thi có kết
quả đúng ở bài tốn này.


Cẹu 14.Ta thÊy quy luẺt lộ sè viạt bến dđắi bỪng
hiỷu hai sè phÝa trến sè ệã. Do ệã x 2.


Nhận xét: Bài toán này khá quen thuộc, có


63,79% số thí sinh dự thi cú kt qu ỳng.


Câu 15.


Gọi N là trung điểm của AD thì


Vì tam giác ABC cân có M là trung điểm của AB
nên CM AB.


Ta có MN lộ ệđêng trung bừnh cựa tam giịc ABD
nến MN BC.


Đặt thì


Ta có hnh thang CDMN l hừnh thang cẹn (vừ cã
hai ệđêng chĐo bỪng nhau).


Suy ra
Do đó


C©u 16.


Ta lẺp hỷ trơc tảa ệé vắi trôc hoộnh AB, trôc tung
AD. Giờ sỏ A(0; 0), B(2013; 0), C(2013; 2014),
D(0; 2014). Con kiạn ệụng ẻ ệửnh A, khi di chuyÓn
theo ệđêng chĐo cựa mét ề vuềng thừ hoộnh ệé
tẽng (hoẳc giờm) ệi mét ệển vỡ vộ tung ệé tẽng
(hoẳc giờm) ệi mét ệển vỡ. Do ệã tảa ệé cựa con
kiạn luền cã dỰng (lĨ; lĨ) hoẳc (chơn; chơn).
VẺy cuèi cỉng con kiạn chử cã thÓ ệạn ệửnh D.



NhẺn xĐt:ậẹy lộ bội toịn tữ hĩp, cịch giời dỰng toịn
nộy bỪng phđểng phịp tảa ệé tá ra rÊt hiỷu quờ, mét
sè thÝ sinh ệở chử ra 1 cịch ệÓ con kiạn bư ệạn D, cã
21,84 % sè thÝ sinh dù thi giời ệđĩc bội toịn nộy.


o x o


90 x x x 36 .
2


o


MCD CDN 90 B B DAB.
x


DAB .
2
A B x


1


ND AD CM.
2


Bµi 1. a) Ta cã


1000T 10T 1956,5656... 19,5656...
1956 19 1937.



b) T ú 990T 1937 nờn


Đáp sè:a) 1937. b)


Nhận xét.Đây là một bài tốn lớp 6 khơng khó.
Có nhiều cách để đổi một số thập phân vơ hạn
tuần hồn về một phân số. Cách trên là một ví dụ.


Bài 2. a) Từ AE a suy ra BE 1 a.
Do đó


b) Vì EFC đều nên EF FC CE.


Suy ra FDC EBC nên DF BE 1 a.
Do đó AF AE a.


áp dụng định lí Pythagore cho các tam giác
vng AEF và BCE, ta có EF AE 2 a 2,


BCE BC.BE 1 a


S .


2 2


1937
T .


990
1937


T .


990


</div>
<span class='text_page_counter'>(26)</span><div class='page_container' data-page=26>

25



Suy ra (1)


hay a2 2a 2 2a2 a2 2(a 1)


Đáp số:a) b)


Nhn xt.Mấu chèt cựa bội toịn lộ thiạt lẺp ệđĩc
phđểng trừnh (1). Nạu giời phđểng trừnh nộy vắi
ệiÒu kiỷn 0 a 1 thừ ta sỳ tÝnh ệđĩc


Từ đó


Suy ra


Tuy nhiên, cách giải này sẽ dài.


Bài 3.a) Ta có (m 2)2 m(m 4)
m2 4m 4 m2 4m 4.


b) Vắi m 2014, ịp dông kạt quờ cẹu a) ta c
20122 2014.2010 4.


Đáp số:a) 4. b) 4.



NhẺn xĐt.ậẹy lộ mét bội toịn tđểng ệèi dƠ.


Bµi 4.a) Ta cã


2014a 8,06 2005,94 8,062
2014a 8,06 (2005,94 8,06)
2014a 8,06 2014


a 8,06.


b) Ta cã 81x2 108xy 36y2 0
9(3x 2y)2 0


Đáp số:a) 8,06. b)


Nhận xét. Đây là bài tốn tính tốn và sử dụng
hằng đẳng thức quen thuộc.


Bội 5. a) Ta thÊy sè chÝnh phđểng ẻ ệẵu cựa dưng
thụ m chÝnh lộ m2.


Từ đó biểu thức ở dịng thứ m là
m2 (m 1)2 m (m 1) 2m 1.
b)


Đáp số:


a) m2 (m 1)2 m (m 1) 2m 1.
b) 96.



NhẺn xĐt. Ta cã thÓ viạt cịc hỪng ệỬng thục tõ
nhọng trđêng hĩp riếng, tõ nhọng sè nguyến
dđểng nhá răi tẽng dẵn. BỪng quy nỰp toịn hảc,
ta cã thÓ tỰo ra nhọng hỪng ệỬng thục tững quịt.
Bội toịn trến lộ mét vÝ dô, tuy khềng khã nhđng
hểi lỰ so vắi cịc thÝ sinh.


Bµi 6.


a) Vì DE // CA nên theo định lí Thales ta có


EA BA BE 6 (cm).
b) Vì FE // GA nên


FG BG BF 2 (cm).


Vì M là trung điểm DE và GM // FE nên G là trung
điểm DF.


Vậy GD GF 2 (cm).


Đáp số:a) 6 cm. b) 2 cm.


Nhận xét.Đây là một bài toán chỉ dùng các tính
chất của định lí Thales để giải và tính tốn sao
cho thật nhanh.


Kạt quờ tững hĩp. Trong tững sè 12 ệiÓm cựa
phẵn thi nộy cựa 29 ệéi, vắi 174 thÝ sinh, mẫi thÝ
sinh lộm 1 cẹu ệđĩc tèi ệa 2 ệiĨm thừ ý 2b) cã


nhiỊu bỰn lộm sai hoẳc khềng giời ệđĩc nhÊt (27
bỰn). Tiạp theo lộ ý 1b) cã 17 bỰn. ý3a) cã Ýt bỰn
giời sai nhÊt lộ 3 bỰn. Cưn lỰi, sè bỰn khềng giời
ệđĩc trong khoờng tõ 4 ệạn 12 bỰn. Cơ thĨ sè
bỰn khềng giời ệđĩc tõng ý nhđ sau:


Bµi 1. a) 10. b) 17.
Bµi 2. a) 13. b) 27.
Bµi 3. a) 3. b) 5.
Bµi 4. a) 9. b) 4.
Bµi 5. a) 7. b) 5.
Bµi 6. a) 6. b) 12.


BF BE 2 <sub>BF</sub> 2.BG <sub>4(cm)</sub>
BG BA 3 3


BE DE 10 2 <sub>BA</sub> 3.BE <sub>18(cm)</sub>
BA CA 15 3 2


m 1 191 1


p 96.


2 2


1.
9
x 1


3x 2y .


6y 9
3 1 2 3 <sub>1.</sub>


2 2 2


2
2


2


a 1 1 1 3 1 3 1
a a 2 2
a


1 1 3 1 <sub>3 1.</sub>
a 3 1 3 1 2


a 1 3.
1.


2
1 a.


2
2


a 1 <sub>1.</sub>
2
a



2


a 2a 2 a 2


2 2 2 2


</div>
<span class='text_page_counter'>(27)</span><div class='page_container' data-page=27>

26


1. Phđểng phịp vẺn dông mèi quan hỷ giọa
ệoỰn thỬng vộ ệđêng gÊp khóc


Ví dụ 1. Cho hình chữ nhật ABCD, E thuộc cạnh
AD. Xác định vị trí các điểm F thuộc cạnh AB, G
thuộc cạnh BC, H thuộc cạnh CD sao cho tứ giác
EFGH có chu vi nhỏ nht.


Lời giải


Gọi I, K, M theo thứ tự là trung điểm của EF, EG
và GH.


Xt AEF vung ti A cã trung tuyạn AI nến
Tđểng tù


Vừ IK lộ ệđêng trung bừnh cựa EFG nến
.


Tđểng tù


Do đó chu vi tứ giác EFGH bằng



EF FG GH HE 2(AI IK KM MC) 2AC.
VËy chu vi tø gi¸c EFGH nhá nhÊt b»ng 2AC khi
và chỉ khi A, I, K, M, C thẳng hàng


EFGH là hình bình hành và EF // BD, FG // AC.


2. Phđểng phịp vẺn dông bÊt ệỬng thục trong
ệđêng trưn


VÝ dô 2. Cho nỏa ệđêng trưn (O; R) ệđêng kÝnh
AB. M lộ ệiÓm di ệéng trến nỏa ệđêng trưn. Qua
M kĨ tiạp tuyạn vắi nỏa ệđêng trưn, gải D, C lẵn
lđĩt lộ hừnh chiạu vuềng gãc cựa A, B trến tiạp
tuyạn Êy. Xịc ệỡnh vỡ trÝ cựa ệiÓm M ệÓ diỷn tÝch tụ
giịc ABCD lắn nhÊt.


Lêi gi¶i


Ta cã AD DC và BC DC nên AD // BC, suy ra
ABCD là hình thang vuông.


V OM DC nn OM // AD vộ O lộ trung ệiÓm AB
nến OM lộ ệđêng trung bừnh cựa hừnh thang
ABCD.


Do đó
Suy ra


H¹ AE BC. Ta có tứ giác ADCE là hình chữ nhật.
Suy ra DC EA.



Vừ nến E thuéc ệđêng trưn ệđêng kÝnh
AB.


Suy ra DC AE 2R (trong ệđêng trưn ệđêng
kÝnh lộ dẹy cung lắn nhÊt).


o
AEB 90


ABCD AD BC


S DC OM DC.
2


AD BC
OM .


2
1


KM EH.
2
1


IK FG
2


1
MC GH.



2
1


AI EF.
2


Một số phương pháp



GIẢI TỐN CC TR HèNH HC



Hà văn nhân


</div>
<span class='text_page_counter'>(28)</span><div class='page_container' data-page=28>

27



Do ú S<sub>ABCD</sub> R.2R 2R2.


DÊu bỪng xờy ra AE lộ ệđêng kÝnh cựa ệđêng
trưn (O) M lộ trung ệiÓm cựa cung AB.


3. Phđểng phịp ịp dông bÊt ệỬng thục ệỰi sè
ệĨ từm cùc trỡ


VÝ dơ 3. Cho nỏa ệđêng trưn (O; R) ệđêng kÝnh
AB. M lộ ệiÓm chuyÓn ệéng trến nỏa ệđêng trưn.
Xịc ệỡnh vỡ trÝ cựa M ệÓ ệỰt giị trỡ lắn
nhÊt.


Lêi gi¶i



Ta cã (gãc néi tÝạp chớn nỏa ệđêng
trưn).


Theo định lí Pytago ta có MA2 MB2 AB2 4R2.
áp dụng bất đẳng thức Bunhiacopxki ta có


Dấu bằng xảy ra tam giác
MAB là nửa tam giác đều


Ví dụ 4. Cho hình vng ABCD có cạnh bằng
12 cm, E là trung điểm của CD, điểm F thuộc cạnh
BC sao cho CF 4 cm. Các điểm G và H theo thứ
tự di chuyển trên các cạnh AB và AD sao cho
GH // EF. Xác định vị trí của điểm G sao cho tứ
giác EFGH có diện tích lớn nhất. Tính diện tích lớn
nhất đó.


Lêi gi¶i


Đặt BG x. Ta có DE EC 6 cm, BF 8 cm.
Ta có AGH đồng dạng với CEF nên


VËy MaxS 75 khi vµ chØ khi x 3.


DiƯn tÝch cđa tø giác EFGH lớn nhất là 75 cm2khi
BG 3 cm.


4. Phng phịp ụng dơng qịy tÝch ệĨ từm cùc trỡ
VÝ dơ 5. Trong cịc tam giịc cã mét cẳp cỰnh bỪng
nhau vộ cẳp gãc ệèi diỷn vắi cẳp cỰnh ệã bỪng


nhau, tam giịc nộo cã


a) DiƯn tÝch lín nhÊt?
b) Chu vi lín nhÊt?


Lêi gi¶i


EFGH ABCD AGH BGF CEF DHE


2
2
2


2 2


AH AG AH 12 x <sub>AH</sub> 2(12 x)<sub>.</sub>


CF CE 4 6 3


2(12 x) 12 2x
DH 12


3 3


S S S S S S


1 2


144 . .(12 x) 4x 12 (12 2x)
2 3



1


144 (144 24x x ) 4x 12 12 2x
3


1


144 48 8x x 6x 24
3


1<sub>x</sub> <sub>2x 72</sub> 1<sub>(x 3)</sub> <sub>75 75.</sub>


3 3


o


s®MA 60 .
3.MA MB


2 2


2


MA 3.MB (1 3)(MA MB )
4.4R 4R.


o
AMB 90



</div>
<span class='text_page_counter'>(29)</span><div class='page_container' data-page=29>

28



XÐt ABC bÊt k× cã BC a, xÐt ®iĨm A
n»m vỊ mét phÝa cđa BC.


Khi đó A nằm trên cung chứa góc dựng trên
đoạn BC.


a) Gäi A’ lµ điểm chính giữa của của cung chứa
góc nói trên.


Kẻ AH BC, A’H’ BC.


Ta cã AH A’H’ nªn S<sub>ABC</sub> S<sub>A’BC</sub>.


Vậy trong các tam giác ABC nói trên, tam giác cân
ở đỉnh A có diện tích lớn nhất.


b)


Trên tia đối của tia AB lấy điểm D sao cho AD AC.
Ta có nên D di chuyển trên cung chứa
góc dựng trên đoạn BC.


Chu vi tam gi¸c ABC lín nhÊt
BA AC lín nhÊt BD lớn nhất.


Tâm của cung chứa góc là điểm chính giữa A
của cung chứa góc .



Gọi giao điểm cđa BA’ víi cung chøa gãc lµ D’.


Ta cã BD BD’ (ệđêng kÝnh lộ dẹy lắn nhÊt cựa
ệđêng trưn).


Do đó AB AC BA’ A’C.


VËy trong c¸c tam ABC nói trên thì tam giác cân
tại A có chu vi lín nhÊt.


Bµi tËp


Bài 1. Cho hình vng ABCD có AB 6 cm, điểm
E nằm trên cạnh AB sao cho AE 2 cm. Hình
thang EFGH có G thuộc cạnh CD, H thuộc cạnh
AD, F thuộc cạnh BC và EH // GF // BD. Xác định
vị trí điểm F trên cạnh BC sao cho hình thang
EFGH có diện tích lớn nhất.


Bội 2. Cho ệoỰn thỬng AB cã ệé dội 2a. Vỳ vÒ mét
phÝa cựa AB cịc tia Ax vộ By vuềng gãc vắi AB.
Qua trung ệiÓm M cựa AB cã hai ệđêng thỬng
thay ệữi luền vuềng gãc vắi nhau vộ cớt Ax, By
theo thụ tù ẻ C, D. Xịc ệỡnh vỡ trÝ cựa cịc ệiÓm C,
D sao cho tam giịc MCD cã diỷn tÝch nhá nhÊt.


Bội 3. Cho tam giịc ABC (AB AC). LÊy ệiÓm D
trến cỰnh BC (D khịc B vộ C). Gải r<sub>1</sub>, r<sub>2</sub> lẵn lđĩt
lộ bịn kÝnh ệđêng trưn néi tiạp cịc tam giịc ABD
vộ ACD. Xịc ệỡnh vỡ trÝ cựa D ệÓ tÝch r<sub>1</sub>r<sub>2</sub> ệỰt giị


trỡ lắn nhÊt.


2
2


2
BDC


2


</div>
<span class='text_page_counter'>(30)</span><div class='page_container' data-page=30>

29



Cho tam giác ABC có


Tia AC cắt trung trực của BC ở E.
Vui băn khoăn không biết trong hai
đoạn thẳng BC và AE đoạn nào dài
hơn. Các bạn chọn đoạn nào?


phạm tuấn khải


(29/67 ậđêng Giịp Bịt, Hoộng Mai,
Hộ Néi)


o o
A 30 , B 10 .


TÍNH ĐƯỢC KHƠNG?

(TTT2 sè 134)



ĐOẠN NÀO DÀI HƠN?




Ta cã


Gải H lộ chẹn ệđêng cao hỰ tõ A xuống BC.


Vì B, C là các góc nhọn nên H thuộc cạnh BC.
Vì nên tam giác ABH vuông cân tại H.
Suy ra HB HA. (1)


Vì tam giác HAC vuông tại H và E là trung điểm
AC nên EA EC EH.


Do ú cỏc tam giác EAH, ECH cân tại E.
Suy ra


Vậy tam giác EAH đều.
Suy ra HA HE. (2)


Từ (1) và (2) suy ra HB HE.
Do đó


VËy


Từ đó ta thấy Vui nói đúng.


NhẺn xĐt. RÊt nhiỊu bỰn ệở gỏi lêi giời vÒ tưa
soỰn. TÊt cờ cịc bỰn ệỊu giời ệóng. Cịc bỰn sau
ệẹy ệđĩc tẳng quộ: NguyÔn Minh TuÊn, 7B, THPT
chuyến Hộ Néi - Amsterdam, Hộ Néi; Tõ Quang
Bừnh, 7C8, THCS ậộ Nơng, Ngề QuyÒn, Hời


Phưng; NguyÔn Thanh Sển, 6/1, THCS NguyÔn
Khuyạn, ậộ Nơng; NguyÔn Hoộng Hđêng, 7A2,
THCS HỰ Hưa, HỰ Hưa, Phó Thả; ậẳng Quang
Anh, 7A, THCS Ngun ChÝch, ậềng Sển, Thanh
Hãa.


anh com pa


o
AEB EBH C 45 .


o


1 1


HBE EHC ACH 15 .
2 2


o
AEH 2ECH 60 .


o
B 45


o o


</div>
<span class='text_page_counter'>(31)</span><div class='page_container' data-page=31>

30


NhẺn xĐt.ậa sè cịc bội gỏi vÒ ệÒu chử ra lêi giời sai
ẻ bđắc tịch



v× chða cã ®iỊu kiƯn a, b 0.


Lêi giời ệóng.ậiỊu kiỷn cựa bội ra lộ ab 0, ta
chia ra cịc trđêng hĩp sau:


Trđêng hĩp 1.a, b 0, giời nhđ bội ra.


Trđêng hĩp 2.a, b 0. Ta cã


Trđêng hĩp 3.ab 0, bỰn ệảc tù giời.


Cịc bỰn ệđĩc nhẺn thđẻng kừ nộy: Nhãm bỰn
Hoộng MỰnh Tđêng, Hoộng Thỡ Khịnh Nguyến,
Trẵn Nhẹn Khiếm, 9C, THCS Hoộng Xuẹn Hởn,
ậục Thả, Hộ Tỵnh; Hă NhẺt Quang, 7A, THCS
Cao Xuẹn Huy, DiÔn Chẹu, Nghỷ An; ậẳng
Thanh Tỉng, 8B, THCS Ngun Thđĩng HiỊn,
ụng Hưa, Hộ Néi.


anh kÝnh lóp


2 2


2a ab 6b


2( a) 3 ( a)( b) 4 ( a)( b) 6( b)
a(2 a 3 b) 2 b(2 a 3 b)
( a 2 b)(2 a 3 b).


ab a. b


a a. a, b b. b,


Trong một giờ học nhóm,
Hồng đố Hà giải bài tốn
sau.


Bội toịn. Cho tam giịc
ABC cẹn tỰi A. KĨ ệđêng
thỬng xy qua A sao cho M lộ ệiÓm
nỪm giọa A vộ C. Chụng minh rỪng tia BM ct
ờng thng xy.


Hà giải nh sau.


Lời giải.Kẻ AH BC.


Theo giờ thiạt tam giịc ABC cẹn tỰi A nến AH lộ
ệđêng phẹn giịc cựa


Mà nên xy AH.
Do đó xy // BC. (1)


Vì M nằm giữa A và C nên
Do đó BM cắt BC tại B. (2)


Từ (1) và (2) suy ra đpcm.


Xem xong lời gii cựa Hộ, Hăng vÉn cờm thÊy
cưn thÊy cã ệiÒu gừ ệã chđa ữn nhđng khềng chử
ra ệđĩc ệã lộ ệiÒu gừ. BỰn cã thĨ chử ra ệiỊu chđa


ữn ệã lộ g khng?


Phạm Liên


(GV. THCS Mai Dịch, Cầu Giấy, Hà Nội)


o o


0 MBC ABC 180 .


xAH yAH
xAB yAC


BAC.


xAB yAC.


(TTT2 sè 133)


CÒN CẦN GÌ NỮA KHƠNG?



</div>
<span class='text_page_counter'>(32)</span><div class='page_container' data-page=32>

31



trđểng cềng thộnh

(sđu tẵm)



(TTT2 số 134)


Nhận xét. Tất cả các bạn tham gia gửi bài đều


chản ệịp ịn ệóng lộ E, nhđng khềng nếu ệđĩc
chÝnh xịc quy luẺt. Cịc bỰn mắi chử ra quy luẺt


cựa hộng thụ nhÊt vộ hộng thụ ba, khềng ệÒ cẺp
ệạn quy luẺt cựa hộng thụ hai.


Quy luật. Ba hình ở cột giữa có trục đối xứng
chung. Trong mỗi hàng, các hình ở cột thứ nhất
và cột thứ ba đối xứng nhau qua trục nói trên. Vậy
hình chèn vào dấu hỏi chấm là hình E.


Chử cã ba bỰn ệđĩc trao thđẻng kừ nộy: NguyÔn
Minh TuÊn, 7B, THPT chuyến Hộ Néi
-Amsterdam, Hộ Néi; NguyÔn ậục ThuẺn, 9A3,


THCS L©m Thao, L©m Thao, Phó Thä; Ph¹m
Thïy Linh, 9C, THCS Nam Cao, Lý Nhân, Hà
Nam.


nguyễn xuân bình


HèNH NAỉO NG?



QUAN ST VAỉ TNH TON


Bài 1.Từ các hình A, B, C, D


Chản hừnh ệóng ệĨ chÌn vộo dởy cịc hừnh di y.


Bài 2.Điền số còn thiếu vào dấu ...


Cc bỰn ệđĩc thđẻng kừ nộy: NguyÔn Danh
Lẹn, 7/2, THCS NguyÔn Khuyạn, CÈm Lỷ, ậộ
Nơng;Phỉng Diỷu Linh, 7B7, THCS LỰc Viến,


Ngề QuyÒn, Hời Phưng; ậẫ Minh Hiỷp, 7A,
THCS Vỵnh Yến, Vỵnh Yến, Vỵnh Phóc;Bỉi Tề
Yến Bừnh, 8A6, THCS Thỡ trÊn Cựng Sển, Sển
Hưa,Phó Yến;Ngun Vẽn Quang, 7A, THCS
Lế Vẽn Thỡnh, Gia Bừnh, Bớc Ninh.


Đặc biệt bạn Nguyễn Văn Quang đã giải cờ
bằng bài thơ nhð sau:


ThÕ cê qu¶ thËt rÊt hay


Tđẻng chõng rÊt dƠ nhđng mộ khã ghế
Ta phời suy nghỵ say mế


Bộn ệi tÝnh lỰi thạ thừ mắi ra
Tđắng ta xuÊt nđắc trđắc tiến
Khềng cẵn xe trđắc chỬng cẵn ệạn ai


Tđắng ta ệang ệụng e2
F1 sn sng i nc bn en


Tịnh đen thấy thế đi liền
Đang từ g1 lên liền f2


Tng trng thấy mất qun xe
ang từ f1 ln liền f2


Đến đây bên đen thua rồi
Vua đi đâu cũng bị vồ chết luôn.



Lê thanh tĩ

THẾ CỜ



Trắng đi trước chiếu hết sau 2 nước.


LEÂ THANH TÚ


</div>
<span class='text_page_counter'>(33)</span><div class='page_container' data-page=33>

32



THÁP NGHIÊNG



D

O

M

I

N

O



GS.Ngun TiÕn Dịng


(Bµi viÕt với sự hợp tác của Sputnik Education)


</div>
<span class='text_page_counter'>(34)</span><div class='page_container' data-page=34>

33



By giê ta thỏ hừnh dung dỉng cịc quẹn domino ệẳt xạp lến nhau tõng viến mét ệÓ xẹy thộnh mét cịi
thịp nghiếng (mẫi tẵng cã 1 viến domino), sao cho nã khềng bỡ ệữ. Nã sỳ bỡ ệữ, nạu cã mét viến domino
nộo ệã trong thịp, mộ phẵn phÝa trến nã lỰi cã ệiÓm trảng tẹm nỪm ra phÝa bến ngoội cựa viến ệã khi
chiạu tõ trến xuèng. ậÓ cho ệển giờn, ta sỳ hừnh dung cịc viến domino nhđ lộ cịc hừnh chọ nhẺt cã
cỉng kÝch thđắc, vộ cỉng nỪm ngang, xạp chăng lến nhau.


VÊn ệÒ lộ, lộm sao mộ xạp ệđĩc n㠓rÊt chi lộ nghiếng”, mộ vÉn khềng ệữ? Cơ thĨ hển, cẹu hái thụ nhÊt
lộ, cã thÓ xạp sao cho tẵng trến cỉng nỪm hoộn toộn ra phÝa ngoội cựa tẵng ệẵu tiến khi chiạu tõ trến
xuèng ệđĩc khềng?


1) Nạu cịi thịp ệã lộ “nghiếng ệÒu”, tục lộ cụ viến trến chỷch ra so vắi viến dđắi cỉng mét ệoỰn, thừ cã


thÓ xẹy sao cho viến trến cỉng nỪm thưi ra hỬn bến ngoội so vắi viến dđắi cỉng mộ khềng bỡ ệữ khềng?
2) Bá ệi ệiÒu kiỷn nghiếng ệÒu, viến nộo muèn thưi ra tõng nộo thừ thưi, thừ cã ệđĩc khềng? Nạu ệđĩc
thừ cẵn Ýt nhÊt bao nhiếu viến?


3) Nạu cã n viến, thừ viến trến cỉng thưi ra ệđĩc so vắi viến dđắi cỉng nhiÒu nhÊt lộ bao nhiếu (ệé thưi
ra cùc ệỰi bỪng bao nhiếu lẵn ệé dội cựa mét viến)?


(Surprise surprise: vÒ mẳt “lÝ thuyạt”, nạu khềng hỰn chạ sè viến vộ chiỊu cao, cã thĨ lộm cho thịp
domino thưi ra bến ngoội bao nhiếu còng ệđĩc! Dịng ệiỷu thưi ra cựa thịp nghiếng domino “tèi đu” tuẹn
theo mét dởy sè toịn hảc rÊt lộ thó vỡ, cịc bỰn thỏ nghỵ xem lộ dởy sè gừ?)


1.7.1954Thộnh phè Nam ậỡnh, thộnh phè ệẵu
tiến cựa Viỷt Nam ệđĩc giời phãng, nẽm nay trưn
60 nẽm.


20,21.7.1954ậếm 20 rỰng 21.7.1954 Hiỷp ệỡnh
Giểnevể ệđĩc kÝ, lẺp lỰi hưa bừnh ẻ miÒn Bớc
Viỷt Nam, nđắc ta tỰm chia lộm 2 miỊn.


28.7.1995ViƯt Nam gia nhËp ASEAN.


2.8.1964 Hời quẹn Viỷt Nam ệịnh ệuữi tộu
Madox ra khỏi hi phn nc ta.


5.8.1964 Ngày truyền thống của Hải quân Việt
Nam, năm nay tròn 50 năm.


8.8.1967Thnh lp Hip hi các Quốc gia Đông
Namá(Association of Southeast Asian Nations,
viết tắt ASEAN). Hiệp hội đang tiến tới thành lập


Cộng đồng ASEAN vào 12.2015.


19.8.1945Cịch mỰng thịng Tịm tiạn tắi thộnh
lẺp nđắc Viỷt Nam Dn ch Cộng ha.


VKT


</div>
<span class='text_page_counter'>(35)</span><div class='page_container' data-page=35>

34



DTH(Dịch và giới thiệu)


thi



OLYMPIC TOÁN HỌC TRẺ QUỐC TẾ



BULGARIA (BIMC 2013)



B. Đề thi đồng đội


1.Mét trang trỰi găm 5 5 ề vuềng trong ệã cã
nhộ cựa 6 nềng dẹn A, B, C, D, E vộ F nhđ hừnh
vỳ dđắi ệẹy. 19 ề vuềng cưn lỰi ệđĩc chia cho
nhọng ngđêi nềng dẹn ệã. Nềng dẹn D sỳ nhẺn
ệđĩc 5 ề vuềng, nềng dẹn A vộ F mẫi ngđêi sỳ
nhẺn ệđĩc 4 ề vuềng, cịc nềng dẹn B, C vộ D mẫi
ngđêi sỳ nhẺn ệđĩc 2 ề vuềng. Cịc nềng dẹn chử
nhẺn ệđĩc cịc ề vuềng trến cỉng hộng, hoẳc
cỉng cét chụa ề vuềng cã nhộ cựa hả vộ cịc ề
vuềng cựa hả phời ệđĩc kạt nèi tắi ngềi nhộ cựa
hả qua cịc ề vuềng mộ ngđêi nềng dẹn ệã nhẺn


ệđĩc. Trong hừnh vỳ sau, hởy ệiÒn A, B, C, D, E
hoẳc F vộo cịc ề vuềng cưn lỰi ệÓ thÓ hiỷn nhọng
ngđêi nềng dẹn nhẺn ệđĩc ề vuềng nộo. (Canada
ệÒ nghỡ)


Meifeng viạt mét cẹu chuyỷn ngớn trong 5
ngộy. Sè cịc chọ mộ cề Êy viạt mẫi ngộy lộ sè
nguyến dđểng. Mẫi buữi tèi cề Êy ệạm sè chọ mộ
cề Êy ệở viạt kÓ tõ ngộy ệẵu tiến ệạn thêi ệiÓm ệã.
Cề Êy lÊy sè thu ệđĩc cựa ngộy thụ nhÊt chia cho
1 2, sè thu ệđĩc cựa ngộy thụ hai chia cho 2 3,
sè thu ệđĩc cựa ngộy thụ ba chia cho 3 4, sè thu


ệđĩc cựa ngộy thụ tđ chia cho 4 5 vộ sè thu
ệđĩc cựa ngộy thụ nẽm chia cho 5 6. Tững cịc
kạt quờ cã ệđĩc cựa 5 phĐp tÝnh ệã lộ 5. Từm sè
cịc tõ ngớn nhÊt cã thĨ trong cẹu chuyỷn ngớn
cựa Meifeng. (Indonesia ệỊ nghỡ)


3.Hừnh vỳ dđắi ệẹy thĨ hiỷn 9 hừnh trưn, cịc hừnh
trưn kỊ nhau tiạp xóc vắi nhau. Mét hừnh trưn ệđĩc
dịn nhởn 1. Cịc hừnh trưn khịc ệđĩc dịn nhởn 1,
2, 3, 3, 3, 4, 4 vộ 4 sao cho khềng cã hai hừnh trưn
tiạp xóc vắi nhau cã nhởn gièng nhau. Hái cã bao
nhiếu cịch dịn nhởn nhđ vẺy? (Trung Quèc ệÒ
nghỡ)


</div>
<span class='text_page_counter'>(36)</span><div class='page_container' data-page=36>

35


5.Mét cỏa hộng cã 350 ệă lđu niỷm vắi cịc mục
giị tđểng ụng lộ 1, 2, 3, ... vộ 350 ệề la. Daniela

cã 50 ệăng 2 ệề la vộ 50 ệăng 5 ệề la ngoội ra
khềng cã ệăng tiÒn nộo khịc. Cề Êy muèn mua
mét mãn ệă lđu niỷm vộ khỬng ệỡnh chử trờ chÝnh
xịc mét sè ệăng tiÒn (Khềng bao giê thay ệữi). Cã
bao nhiếu trong sè 350 ệă lđu niỷm mộ cề Êy cã
thĨ chản? (Romania ệỊ nghỡ)


6.Tững cựa 1997 sè nguyến dđểng lộ 2013. TÝnh
hiỷu giọa giị trỡ lắn nhÊt vộ giị trỡ nhá nhÊt cựa
tững cịc bừnh phđểng cựa cịc sè ệã. (Trung Quèc
ệÒ nghỡ)


7. Sè 16 ệđĩc ệẳt ẻ gãc bến trịi cựa mét bờng
4 4 nhđ hừnh vỳ. 15 ề vuềng cưn lỰi chử ệiÒn mét
trong cịc sè 1 , 2, ..., 15 sao cho tững cựa bèn sè
trong cịc hộng, cịc cét, cịc ệđêng chĐo ệỊu
bỪng nhau. Từm giị trỡ lắn nhÊt cã thĨ cựa tững 6
sè trong cịc ề ệđĩc tề mộu trong hừnh vỳ.
(Romania ệÒ nghỡ)


Hai ề vuềng ẻ gãc cựa mét bờng chọ nhẺt 7 8 ệđĩc bá ệi theo ba cịch nhđ hừnh vỳ dđắi ệẹy. Ta
sỳ chia phẵn cưn lỰi thộnh 18 hừnh chọ nhẺt cã kÝch thđắc 1 3 hoẳc 3 1. Chụng minh rỪng cã mét
trong ba hừnh ệở cho khềng thĨ chia ệđĩc. (Canada ệỊ ngh)


</div>
<span class='text_page_counter'>(37)</span><div class='page_container' data-page=37>

36


Bài I.(2,0 điểm)


1) Tính giá trị của biĨu thøc khi x 9.
2) Cho biĨu thøc



víi x 0 vµ x 1.


a) Chứng minh rằng
b) Tìm các giá trị của x để


Bội II. (2,0 ệiÓm) Giời bội toịn sau bỪng cịch
lẺp phđểng trừnh hoẳc hỷ phđểng trừnh:


Mét phẹn xđẻng theo kạ hoỰch cẵn phời sờn
xuÊt 1100 sờn phÈm trong mét sè ngộy quy ệỡnh.
Do mẫi ngộy phẹn xđẻng ệã sờn xuÊt vđĩt mục
5 sờn phÈm nến phẹn xđẻng ệở hoộn thộnh kạ
hoỰch sắm hển thêi gian quy ệỡnh 2 ngộy. Hái
theo kạ hoỰch, mẫi ngộy phn xng phi sn
xuất bao nhiu sn phẩm?


Bài III.(2,0 điểm)


1) Giời hỷ phđểng trừnh


2) Trến mẳt phỬng tảa ệé Oxy cho ệđêng thỬng
(d): y x 6 vộ parabol (P): y x2.


a) Tìm tọa độ các giao điểm của (d) và (P).
b) Gọi A, B là hai giao điểm của (d) và (P). Tính
diện tích tam giác OAB.


Bài IV.(3,5 điểm)


Cho ờng trn (O; R) cã ệđêng kÝnh AB cè


ệỡnh. Vỳ ệđêng kÝnh MN cựa ệđêng trưn (O; R)
(M khịc A, M khịc B). Tiạp tuyạn cựa ệđêng trưn
(O; R) tỰi B cớt cịc ệđêng thỬng AM, AN lẵn lđĩt
tỰi cịc ệiĨm Q, P.


1) Chøng minh tø gi¸c AMBN là hình chữ nhật.


2) Chng minh bốn iểm M, N, P, Q cỉng thuéc
mét ệđêng trưn.


3) Gải E lộ trung ệiÓm cựa BQ. ậđêng thỬng
vuềng gãc vắi OE tỰi O cớt PQ tỰi ệiÓm F. Chụng
minh F lộ trung ệiÓm cựa BP vộ ME // NF.
4) Khi ệđêng kÝnh MN quay quanh tẹm O vộ
tháa mởn ệiÒu kiỷn ệÒ bội, xịc ệỡnh vỡ trÝ cựa
ệđêng kÝnh MN ệÓ tụ giịc MNPQ cã diỷn tÝch
nhá nhÊt.


Bài V.(0,5 điểm)


Vi a, b, c l cc số dng tháa mởn ệiÒu kiỷn
a b c 2. Từm giị trỡ lắn nhÊt cựa biÓu thục


Q 2a bc 2b ca 2c ab.


4 1 <sub>5</sub>
x y y 1


1 2 <sub>1.</sub>
x y y 1



2P 2 x 5.
x 1.


P
x
x 2 1 <sub>.</sub> x 1
P


x 2 x x 2 x 1


x 1
A


x 1


ĐỀ THI TUYN SINH LP 10


THPT TP. HAỉ NI



Năm học: 2014 - 2015



</div>
<span class='text_page_counter'>(38)</span><div class='page_container' data-page=38>

37


Bài 1.(2 điểm)


Gii cc phng trừnh vộ hỷ phđểng trừnh sau:
a) x2 7x 12 0


b)


c) x4 9x2 20 0


d)


Bài 2. (1,5 điểm)


a) V th (P) cựa hộm sè y x2vộ ệđêng thỬng
(D): y 2x 3 trến cỉng mét hỷ trôc tảa ệé.
b) Từm tảa ệé cịc giao ệiÓm cựa (P) vộ (D) cu
trn bng php tính.


Bài 3. (1,5 điểm)


Thu gọn các biểu thức sau:


(với x 0).


Bài 4.(1,5 điểm)


Cho phng trnh x2 mx 1 0 (1) (x lộ Èn sè).
a) Chụng minh phđểng trừnh (1) luền cã hai
nghiỷm trịi dÊu.


b) Gải x<sub>1</sub>, x<sub>2</sub>lộ cịc nghiỷm cựa phđểng trừnh (1).
TÝnh giị trỡ cựa biÓu thc


Bài 5.(3,5 điểm)


Cho tam gic ABC cã ba gãc nhản, néi tiạp
ệđêng trưn tẹm O (AB AC). Cịc ệđêng cao AD
vộ CF cựa tam giịc ABC cớt nhau tỰi H.



a) Chụng minh tụ giịc BFHD néi tiạp. Suy ra
b) Gải M lộ ệiÓm bÊt kừ trến cung nhá BC cựa
ệđêng trưn (O) (M khịc B vộ C) vộ N lộ ệiÓm ệèi
xụng cựa M qua AC. Chụng minh tụ giịc AHCN
néi tip.


c) Gọi I là giao điểm của AM và HC, J là giao
điểm của AC và HN. Chứng minh


d) Chøng minh r»ng: OA vu«ng gãc víi IJ.AJI ANC.


o


AHC 180 ABC.


2 <sub>2</sub>


1 1 <sub>2</sub> <sub>2</sub>


1 2


x <sub>x 1 x x 1</sub>


P .


x x


x 1 2 6


B : 1



x 3 x x 3 x x 3 x
5 5 5 3 5


A ,


5 2 5 1 3 5
3x 2y 4


4x 3y 5.
2


x ( 2 1)x 2 0


ĐỀ THI TUYỂN SINH LỚP 10 THPT TP. HỒ CHÍ MINH



Nhân kỉ niệm 40 năm đội tuyển Việt Nam dự thi
Olympic toán Quốc tế 1974-2014, ngày 10.8.2014
tại TP. Vĩnh Yên, Vĩnh Phúc, Sở GD&ĐT Vĩnh
Phúc phối hợp với Hội Toán học Hà Nội tổ chức
Hội thảo khoa học: Một số chuyên đề Toán học
qua 40 năm Việt Nam tham dự thi Olympic tốn
Quốc tế. Đến dự Hội thảo có: TS. Vũ Đình Chuẩn,


Vơ trđẻng Vơ GDTrH; TS. Mai Vẽn Trinh, Côc
trđẻng Cơc Khờo thÝ vộ KiĨm ệỡnh chÊt lđĩng Giịo
dơc; GS.TSKH Trẵn Vẽn Nhung, Tững thđ kÝ Héi
ệăng chục danh Giịo sđ nhộ nđắc, Bé GD&ậT;
GS.TSKH. NGND. NguyÔn Vẽn MẺu, Chự tỡch Héi
Toịn hảc Hộ Néi; ThS. Hoộng Minh Quẹn, Giịm


ệèc Sẻ GD&ậT Vỵnh Phóc; ThS. Ngun Họu ậé,
Giịm ệèc Sẻ GD&ậT Hộ Néi; ThS. Vò Kim Thựy,
Phã Tững Thđ kÝ Héi TH Hộ Néi; Tững biến tẺp tỰp
chÝ Toịn Tuữi thể; TSKH. Vò ậừnh Hưa, Giờng viến
ậỰi hảc Sđ phỰm Hộ Néi vộ cịc GS, TS. Cịc nhộ
giịo ệở vộ ệang tham gia băi dđìng hảc sinh giái
mền toịn. Trong héi thờo ệở cã nhiÒu bội tham
luẺn ệịnh giị vÒ thộnh tÝch ệéi tuyÓn Viỷt Nam
tham gia dù thi Olympic toịn Quèc tạ trong 40
nẽm qua vộ cịc bội tham luẺn vÒ chuyến mền
toịn ệđĩc trừnh bộy. TỰp chÝ Toịn Tuữi thể tẳng
quộ cho cịc ệỰi biĨu dù Héi thờo.


PV


HỘI THẢO KHOA HỌC



MỘT SỐ CHUN ĐỀ TỐN HỌC QUA 40 NĂM VIT NAM THAM D OLYMPIC TON QUC T



Năm học: 2014 - 2015



</div>
<span class='text_page_counter'>(39)</span><div class='page_container' data-page=39>

38



Bài 1(134). Tìm số dð trong phÐp chia cña sè
A (2014 1)(2014 2)(2014 3)... (2014 4028)
cho 32014.


Lêi gi¶i.Ta cã


A 1.2.3...2014.(2014 1)...(2014 4028)


: (1.2.3...2014)


1.2.4.5.7.8...(3.2014 2)(3.2014 1).3.6.9
...(3.2014) : (1.2.3...2014)


1.2.4.5.7.8...(3.2014 2)(3.2014 1).32014.
Do đó A chia hết 32014.


VËy sè dð trong phÐp chia A cho 32014lµ 0.


Nhận xét. Đây là một bài tốn khơng khó nên có
rất nhiều bạn tham gia giải bài. Hầu hết các bạn
tham gia giải bài đều giải đúng, một số bạn biến
đổi phức tạp. Các bạn sau có lời giải đúng:


Ngun Minh §øc, 6C, THCS Ngun Cao, Q
Vâ; Ngun ThÞ Ngäc nh, 6A3, THCS Yên
Phong, Yên Phong, Bắc Ninh; Lê Thị Vân Anh,
6E; Nguyễn Thị Thảo Linh, Nguyễn Thị Hoàng
Cúc, 7D, THCS Nhữ Bá Sỹ, Hoằng Hóa; Đặng
Quang Anh, 7A, THCS Nguyễn Chích, Đông Sơn,


Thanh Hóa; NguyÔn Dđểng Hoộng Anh, 7C,
THCS Vẽn Lang, Viỷt Trừ; NguyÔn Thỉy Dđểng,
6A3, THCS Lẹm Thao, Lẹm Thao; Hoộng Lế
Cềng Khềi, 7B, THCS Thanh Hộ, Thanh Ba, Phó
Thả; Ngun Tiạn Dịng, 6D5, THCS Trẵn Phó,


Hời Phưng; Hă Xuẹn Viỷt Anh, 6A, THCS Hă
Xuẹn Hđểng, Quúnh Lđu, Nghỷ An; Lế Ngảc


Hoa, 6E1; Kim Thỡ Hăng Lỵnh, 7E1, THCS Vỵnh
Tđêng, Vỵnh Tđêng; NguyÔn Duy Quý, 7A1, THCS
Lế Hăng Phong; ậẫ Minh Trung, 7A1, THCS Hai
Bộ Trđng, TX. Phóc Yến, Vỵnh Phóc;TỰ Lế Ngảc
Sịng, 7E; NguyÔn Anh ậục, NguyÔn Vẽn ậục,
6D, THPT chuyến Hộ Néi - Amsterdam; KhuÊt
Bờo Chẹu, 7A, THCS ThỰch ThÊt, ThỰch ThÊt, Hộ
Néi;Phan Thỡ Trộ Phđểng, 7C, THCS Xuẹn Diỷu,
Can Léc, Hộ Tỵnh.


phïng kim dung


Bội 2(134). Chụng minh rỪng nạu k lộ mét sè
nguyến thừ 2016k 3 khềng lộ sè lẺp phđểng. (Sè
lẺp phđểng lộ lẺp phđểng cựa mét sè nguyến)


Lêi giời. Trđắc hạt ta cã nhẺn xĐt sau: LẺp


phđểng cựa mét sè nguyến khi chia cho 9 chử cã
cịc sè dđ l 0, 1, 8.


Thật vậy, giả sử a là một số nguyên.


Với a 3m, m thì a3 27m3, chia hÕt cho 9
hay chia 9 dð 0.


Víi a 3m 1, m th×


a3 27m3 27m2 9m 1, chia 9 dð 1.
Víi a 3m 1, m th×



a3 27m3 27m2 9m 1, chia 9 dð 8.


Trẻ lỰi vắi bội toịn.Ta thÊy 2016k 3 chia 9 dđ 3,
vắi mải k nến khềng thÓ lộ sè lẺp phđểng.


NhẺn xĐt.TÊt cờ cịc lêi giời gỏi vÒ Tưa soỰn ệỊu
ệóng theo nhọng cịch khịc nhau khi sỏ dông
ệăng dđ theo mod 3, mod, 7, mod 9. Sau ệẹy lộ
nhọng bỰn cã lêi giời gản hển cờ: NguyÔn Vẽn
ậục, 6D; TỰ Lế Ngảc Sịng, 7E, THPT chuyến Hộ
Néi - Amsterdam; KhuÊt Bờo Chẹu, 7A, THCS
ThỰch ThÊt, ThỰch ThÊt, Hộ Néi;NguyÔn Phđểng
Hoa, 7A1, THCS Hăng Bộng, Hăng Bộng, Hời
Phưng; Kim Thỡ Hăng Lỵnh, 7E1, THCS Vỵnh
Tđêng, Vỵnh Tđêng, Vỵnh Phóc; Ngun Minh
ậục, 6C, THCS NguyÔn Cao, Quạ Vâ; NguyÔn TrÝ
Thộnh, NguyÔn Thỡ Ngảc nh, 6A3, THCS Yến
Phong, Yến Phong, Bớc Ninh;Vị Linh Chi, 6A1;


Ngun Thu HiỊn, 6A3; TrÇn Qc LËp, 7A3,
THCS Lâm Thao, Lâm Thao, Phú Thọ; Nguyễn
Thị Hoàng Cúc, 7D, THCS Nhữ Bá Sü, Ho»ng
Hãa, Thanh Hãa; Phan Trung HiÕu, 7A, THCS
Cao Xu©n Huy, DiƠn Ch©u, NghƯ An.


Hå quang vinh


Bội 3(134).Giời phđểng trừnh



Lêi giời.ậiỊu kiỷn: 0 x 8.
Ta viạt lỰi phđểng trừnh dđắi dỰng


TH1.


(tháa m·n).
TH2. x 2 8 x x


8 x 2 8 x 4 x 4
x 2 x 8 x


2 2


x 4 x 4 x 2 x(8 x) (8 x)
( x 2) ( x 8 x) .


2


</div>
<span class='text_page_counter'>(40)</span><div class='page_container' data-page=40>

39



ậiÒu kiỷn Ta ệđĩc


(5x 12)2 16(8 x) 25x2 104x 16 0
(x 4)(25x 4) 0 x 4 (do ệiÒu kiỷn).
VẺy phđểng trừnh cã mét nghiỷm lộ x 4.


NhẺn xĐt.ậa sè cịc bỰn gỏi lêi giời ệỊu ệđa ra ệịp
sè ệóng. Mét sè bỰn ệở mớc lẫi khềng ệẳt ệiÒu kiỷn
hoẳc khềng kiĨm tra ệiỊu kiỷn cựa nghiỷm. Sau
ệẹy lộ cịc bỰn cã lêi giời tèt: NguyÔn Nam Viỷt, 8A,


THPT chuyến Hộ Néi - Amsterdam; Ngun Tiạn
ậỰt, ậẳng Thanh Tỉng, 8B, THCS Ngun Thđĩng
HiỊn,ụng Hưa; NguyÔn Thộnh Vinh, 8H, THCS Lế
Quý ậền, Cẵu GiÊy, Hộ Néi;NguyÔn Viỷt Hoộng,
Lế Thỡ Phđểng Thanh, NguyÔn Thỡ Thờo Phđểng,
8A, THCS ậục Lý; Ngun Thóy HỪng, 8A; Trẵn
Thỡ Thu Hộ, 8B, THCS Phó Phóc, Lý Nhẹn, Hộ
Nam; NguyÔn Thỡ HỪng, 8B, THCS Lý NhẺt
Quang, ậề Lđểng, Nghỷ An; ậẳng Quang Anh,
7A, THCS NguyÔn ChÝch, ậềng Sển, Thanh Hãa;


Lế Quang Trung, 8A4, THCS GiÊy Phong Chẹu,
Phỉ Ninh; NguyÔn Dđểng Hoộng Anh, 7C; Lế
Thanh Tỉng, Lế Hỉng, Hă Quang Huy, 8A, THCS
Vẽn Lang, Viỷt Trừ, Phó Thả.


ngun anh dịng


Bài 4(134).Cho a, b, c là độ dài ba cạnh của một
tam giác v tha món 2c b abc.


Tìm giá trị nhỏ nhÊt cđa biĨu thøc


Lêi giời. Vừ a, b, c lộ ba cỰnh cựa mét tam giịc
nến a b c 0, b c a 0, c a b 0.
ịp dông bÊt ệỬng thục (vắi x, y 0)
ta ệđĩc


Từ giả thiết 2c b abc suy ra
Do ú



Đẳng thức xảy ra khi và chØ khi


Vậy S đạt giá trị nhỏ nhất là khi và chỉ khi


NhẺn xĐt.ậẹy lộ mét bội toịn hay vộ khã nến cã
rÊt Ýt bỰn tham gia giời bội. Hẵu hạt cịc bỰn ệÒu
ệđa ra ệđĩc ý tđẻng nhđ trến, mét sè bỰn biạn ệữi
phục tỰp. Nhọng bỰn sau ệẹy cã lêi giời ệóng:


Ngun Nam Viỷt, 8A, THPT chuyến Hộ Néi
-Amsterdam; NguyÔn Thộnh Vinh, 8H, THCS Lế
Quý ậền, Cẵu GiÊy; PhỰm Thỡ Minh Lý, 8A1,
THCS Trđng Vđểng, Thanh Lẹm, Mế Linh, Hộ
Néi;ậẳng Quang Anh, 7A, THCS NguyÔn ChÝch,
ậềng Sển, Thanh Hãa; NguyÔn Thỡ Thanh
Hđểng, 9A; ậẳng Cềng Toịn, 8A, THCS Yến
Phong, Yến Phong, Bớc Ninh; Trẵn Thỡ Thu Hộ,
8B; NguyÔn Thóy HỪng, 8A; Trẵn nh Vẹn, 9A,
THCS Phó Phóc, Lý Nhẹn; Phan Trang Nhung,
9B, THCS Bc Lý, Lý Nhn, H Nam.


Cao Văn dũng


Bi 5(134).Hiỷu ệèi xụng cựa hai tẺp hĩp A vộ B
ệđĩc ệỡnh nghỵa nhđ sau


A B (A B) \ (A B)


tøc là gồm những phần tử hc thc A, hc


thc B nhðng không thuộc cả hai tập.


Chứng minh rằng A B (A \ B) (B \ A).


Lời giải.* TH1. Nếu A B là tập rỗng thì A B.
Khi A B thì (A B) \ (A B) là tập rỗng nên
A B (A \ B) (B \ A) (vì cùng là tập rỗng).
* TH2. Nếu A B không là tập rỗng thì A B.
Khi đó (A \ B) (B \ A) không là tập rỗng.
Lấy phần tử x bất kì thuộc tập A B.
Ta có x A B x (A B) \ (A B)


Do đó A B (A \ B) (B \ A).


NhẺn xĐt.ậẹy lộ bội toịn cể bờn vÒ chụng minh
hai tẺp hĩp bỪng nhau nến cã nhiÒu bỰn gỏi bội
ệạn tưa soỰn, hẵu hạt cịc bỰn lộm tèt trđêng hĩp
A B khịc rẫng. Tuy nhiến cã Ýt bỰn xĐt trđêng
hĩp A B lộ tẺp rẫng. Mét sè bỰn trừnh bộy lêi giời
bỪng phĐp kĐo theo ( ) nhđng quến khềng
chụng minh chiÒu ngđĩc lỰi.


Cịc bỰn sau cã lêi giời tèt hển cờ: NguyÔn Thỡ Mai
Phđểng, NguyÔn Thỡ Hoội Phđểng, Ngun Thỡ
Hun, 6A, THCS Phan Huy Chó, Thanh Hộ, Hộ
Tỵnh;ậẫ Thộnh Cịt Lđĩng, 6A; NguyÔn Thỡ


x A
x A <sub>x A B</sub>
x A B <sub>x B</sub>



x A B <sub>x A B</sub> x B
x A B
x A \ B <sub>x (A \ B) (B \ A).</sub>
x B \ A


a b c 3.


4 3


a b c 3.
1 2 3 3


S 2 2 a 4 3.
c b a a


2 1 a.
b c


1 1 1 1


S 2


b c a a c b b c a a b c
1 1 2 4 6


3


a c b a b c c b a
1 1 4


x y x y


3 4 5


T .


b c a c a b a b c


12 x 8.
5


</div>
<span class='text_page_counter'>(41)</span><div class='page_container' data-page=41>

40


Thanh HuyÒn, ậẫ ậục Thớng, 6B; PhỰm Hoộng
Sển, 7A; NguyÔn Quúnh Anh, 8C, THCS ChÝnh
Lý; NguyÔn Thỡ Thờo Phđểng, Lế Thỡ Phđểng
Thanh, NguyÔn Viỷt Hoộng, 8A, THCS ậục Lý;


TrÇn nh Vân, 9A, THCS Phú Phúc, Lý Nhân, Hà
Nam;Nguyễn Văn Đức, 6D, THPT chuyên Hà Nội
- Amsterdam, Hà Nội; Đặng Quang Anh, 7A,
THCS Nguyễn Chích, Đông Sơn, Thanh Hóa; Bùi
Trâm Anh, 9A, THCS Lập Thạch, Lập Thạch, Vĩnh
Phúc;Hoàng Đức Thuận, 8A, THCS Văn Lang,
TP. Việt Trì, Phú Thọ.


trnh hoi dng


Bi 6(134).Cho tam giịc ABC, ệđêng cao CK, vắi
ệiÓm K thuéc cỰnh AB. Dùng ra phÝa ngoội tam
giịc ABC hai tam giịc CAE, CBF tđểng ụng vuềng


tỰi E, F vộ tháa mởn


Chøng minh r»ng CK2 AE.BF.


Lời giải.


Vì nên ABC có các góc
A, B nhän.


Suy ra ACK CBF, BCK CAE. VËy


NhẺn xĐt.Bội toịn nộy rÊt dƠ, nhiỊu bỰn tham gia
giời. Xin nếu tến mét sè bỰn cã lêi giời tèt: ậinh
Vẽn Hiạu, Lđểng ậộo Quang Anh, Lế Phđểng
Thờo, 9A, THCS Nam Cao, Lý Nhẹn, Hộ Nam;


Hoộng ậục ThuẺn, 8A, THCS Vẽn Lang, TP. Viỷt
Trừ, Phó Thả;Ngun Vẽn ậục, 6D, THPT chuyến
Hộ Néi - Amsterdam; NguyÔn Duy Khđểng, 8A9,
THCS Giờng Vâ, Ba ậừnh, Hộ Néi; NguyÔn Thỡ
Viến, 8A; Ngề Thỡ Quyến, 9A, THCS Yến Phong,
Yến Phong, Bớc Ninh;Phan Vẽn ậỰt, 8C, THCS
Hoộng Xuẹn Hởn, ậục Thả, Hộ Tỵnh; Trẵn Lam
Giang, 8B, THCS Lý NhẺt Quang, ậề Lđểng,


NghƯ An.


Ngun Minh Hµ


2 CK CK BF AE



CK . .CA.CB . .CA.CB AE.BF.
CA CB BC AC


ACE CBA, BCF CAB


ACE CBA, BCF CAB.


Ngun Minh §øc, 6C, THCS Ngun Cao,
Q Vâ; Ngun ThÞ Ngäc nh, 6A3, THCS
Yªn Phong, Yên Phong, Bắc Ninh; Nguyễn
Văn Đức, 6D; Tạ Lê Ngọc Sáng, 7E; Nguyễn
Nam Việt, 8A, THPT chuyên Hà Nội
-Amsterdam; Khuất Bảo Châu, 7A, THCS
Thạch Thất, Thạch Thất; Nguyễn Thành Vinh,
8H, THCS Lê Quý Đôn, Cầu Giấy, Hà Néi;


Kim Thỡ Hăng Lỵnh, 7E1, THCS Vỵnh Tđêng,
Vỵnh Tđêng, Vỵnh Phóc; Ngun Thỡ Hoộng
Cóc, 7D, THCS Nhọ Bị Sủ, HoỪng Hãa;


NguyÔn Thỡ Thờo Phđểng, 8A, THCS ậục Lý;


ậẳng Quang Anh, 7A, THCS NguyÔn ChÝch,
ậềng Sển, Thanh Hãa; Lế Thỡ Thờo Phđểng,
8A, THCS ậục Lý; Trẵn Thỡ Thu Hộ, 8B;


Ngun Thóy HỪng, L Th Phng Thanh, 8A;


Trần nh Vân, 9A THCS Phó Phóc, Lý Nh©n,



Hộ Nam; Ngun Dđểng Hong Anh, 7C;


Hoàng Đức Thuận, 8A, THCS Văn Lang, TP.
Việt Trì, Phú Thọ.


</div>
<span class='text_page_counter'>(42)</span><div class='page_container' data-page=42>

41



KET QUA THI OLYMPIC QUỐC TẾ NĂM 2014



CÁC ĐOÀN CỦA VIỆT NAM



Olympic To¸n Qc tÕ IMO 2014


Kừ thi diƠn ra tỰi thộnh phè Cape Town, Nam Phi
tõ ngộy 3.7 ệạn ngộy 13.7.2014. Cã 560 thÝ sinh
ệạn tõ 111 quèc gia vộ vỉng lởnh thữ tham dù.
ậéi tuyÓn Viỷt Nam găm 6 thÝ sinh do TS. Lế Bị
Khịnh Trừnh lộm trđẻng ệoộn ệở dộnh 3 huy
chđểng Vộng, 2 huy chđểng BỰc vộ 1 huy
chđểng ậăng. Cịc bỰn ệoỰt huy chđểng Vộng
lộ: Trẵn Hăng Quẹn, lắp 12, THPT chuyến tửnh
Thịi Bừnh; PhỰm TuÊn Huy, lắp 12, Phữ thềng
nẽng khiạu, ậỰi hảc Quèc gia TP. Hă ChÝ Minh;
NguyÔn Thạ Hoộn, lắp 11, THPT chuyến ậỰi
hảc KHTN Hộ Néi. Hai bỰn ệoỰt huy chđểng BỰc
lộ Hă Quèc ậẽng Hđng, lắp 12, Phữ thềng nẽng
khiạu, ậỰi hảc Quèc gia TP. Hă ChÝ Minh vộ
Vđểng NguyÔn Thỉy Dđểng, lắp 12, THPT
chuyến Lế Quý ậền, ậộ Nơng. BỰn NguyÔn Huy


Tỉng, lắp 12, THPT chuyến Trẵn Phó, Hời
Phưng ệoỰt huy chđểng ậăng. Xạp hỰng toộn
ệoộn khềng chÝnh thục, ệoộn Viỷt Nam xạp thụ
10 vắi tững sè 159 ệiÓm. Cịc ệoộn cã tững ệiÓm
cao hển lộ: Trung Quèc, Mủ, ậội Loan, Nga,
NhẺt Bờn, Ukraina, Hộn Quèc, Singapore vộ
Canada.


BỰn PhỰm TuÊn Huy tõng ệoỰt huy chđểng
Vộng IMO 2013. BỰn Trẵn Hăng Quẹn tõng ệoỰt
huy chđểng Vộng Olympic Toịn Tuữi thể toộn
quèc nẽm 2007 khi lộ hảc sinh lắp 5 cựa ệoộn
Thịi Bừnh.


Olympic Tin häc Quèc tÕ IOI 2014


Kừ thi diÔn ra tỰi TP. ậội Bớc, ậội Loan, Trung
Quèc tõ ngộy 13.7 ệạn ngộy 20.7.2014. Cã 311
thÝ sinh ệạn tõ 82 quèc gia vộ vỉng lởnh thữ
tham dù. ậoộn Viỷt Nam cã 4 thÝ sinh tham dù
do PGS. TS. NguyÔn Viạt Nghỵa lộm trđẻng
ệoộn. Kạt quờ ệoộn Viỷt Nam ệoỰt 2 huy chđểng
BỰc vộ 2 huy chđểng ậăng. Hai bỰn Ngun
Quang Dịng, lắp 12, THPT chuyến ậỰi hảc Sđ
phỰm Hộ Néi vộ Ngề Hoộng Anh Phóc, lắp 12,
THPT chuyến ậỰi hảc KHTN Hộ Néi ệoỰt giời
BỰc. Hai bỰn ậẫ Xuẹn Viỷt, lắp 12 vộ NguyÔn
Tiạn Trung Kiến, lắp 11, THPT chuyến ậỰi hảc
KHTN Hộ Néi ệoỰt giời ậăng.



Olympic VËt lÝ Quèc tÕ IPhO 2014


Kì thi diễn ra từ ngày 13.7 đến 21.7.2014 tại


TP. Astana, Kazakhstan. Cã 383 thÝ sinh cựa 84
quèc gia vộ vỉng lởnh thữ ệở tham dù. ậoộn Viỷt
Nam tham dù găm 5 thÝ sinh. Kạt quờ, nẽm nay
ệoộn Viỷt Nam ệụng thụ 5, lộ thộnh tÝch cao nhÊt
trong sè nhọng lẵn tham dù vắi 3 huy chđểng
Vộng vộ 2 huy chđểng BỰc. Cịc bỰn ệoỰt huy
chđểng Vộng lộ: Cao Ngảc Thịi, lắp 12, THPT
chuyến Phan Béi Chẹu, Nghỷ An; ậẫ Thỡ BÝch
Huỷ, lắp 12, THPT chuyến ậỰi hảc KHTN Hộ
Néi; Vò Thanh Trung Nam, lắp 11, THPT chuyến
Hộ Néi - Amsterdam, Hộ Néi. Hai bỰn ậộo
Phđểng Khềi, lắp 12, THPT chuyến ậỰi hảc
KHTN Hộ Néi vộ PhỰm NguyÔn TuÊn Anh, lắp
12, Phữ thềng nẽng khiạu, ậỰi hảc Quèc gia
TP. Hă ChÝ Minh ệoỰt huy chđểng BỰc.


Olympic Hãa häc Quèc tÕ IChO 2014


IChO 2014 diÔn ra tõ ngộy 20.7 ệạn ngộy
29.7.2014 ệđĩc tữ chục tỰi Hộ Néi. Cã 291 thÝ
sinh ệạn tõ 77 quèc gia vộ vỉng lởnh thữ tham
dù. ậỰi hảc KHTN Hộ Néi lộ ệển vỡ chự trừ, ậỰi
hảc Sđ phỰm Hộ Néi lộ ệển vỡ phèi hĩp tữ chục
kừ thi nộy. ậoộn Viỷt Nam cã 4 thÝ sinh tham dù
do TS. NguyÔn Quèc ChÝnh, ậỰi hảc Quèc gia
TP. Hă ChÝ Minh lộm trđẻng ệoộn. Bèn thÝ sinh


cựa ệoộn Viỷt Nam tham dù ệÒu ệoỰt giời cao.
Hai bỰn PhỰm Mai Phđểng, PhỰm Ngẹn Giang,
lắp 12, THPT chuyến Hộ Néi - Amsterdam cỉng
ệoỰt huy chđểng Vộng, trong ệã PhỰm Mai
Phđểng lộ mét trong ba thÝ sinh ệỰt ệiÓm cao
nhÊt kừ thi cưn PhỰm Ngẹn Giang nỪm trong 15
thÝ sinh xuÊt sớc nhÊt. Hai thÝ sinh ậẫ Viỷt Hđng,
THPT chuyến Trẵn Phó, Hời Phưng vộ ậoộn
Quèc Hoội Nam, THPT chuyến Quèc hảc Huạ
ệoỰt huy chđểng BỰc.


Olympic Sinh häc Quèc tÕ IBO 2014


Tõ ngộy 5.7 ệạn ngộy 13.7.2014 ệở diÔn ra IBO
2014 tỰi Bali, Indonesia. ậoộn hảc sinh Viỷt
Nam tham dù vắi 4 thÝ sinh ệÒu ệoỰt giời găm 1
huy chđểng BỰc vộ 3 huy chđểng ậăng. Huy
chđểng BỰc thuéc vÒ Lế Thỡ Nguyỷt HỪng,
THPT chuyến Lế Quý ậền, ậộ Nơng. Ba bỰn
ệoỰt huy chđểng ậăng lộ PhỰm Minh ậục, Trẵn
Lế Quèc Khịnh, THPT chuyến Lế Hăng Phong,
Nam ậỡnh; ậộo Trảng Doanh, THPT chuyến
Trẵn Phó, Hời Phưng.


</div>
<span class='text_page_counter'>(43)</span><div class='page_container' data-page=43>

42


Hềm Êy, do nhiÒu viỷc quị nến thịm tỏ
Sếlềcềc rêi cềng ty muén hển mải ngộy. Lóc
ệã ệở gẵn 9 giê tèi nhđng ềng vÉn quyạt ệỡnh
ệi bé vÒ nhộ. Thịm tỏ rÊt thÝch ệi bé, võa ệĨ
rÌn luyỷn sục kháe, võa ệÓ thđ giởn, ngớm

cờnh phè phđêng. ậang rờo bđắc, bÊt chĩt
thịm tỏ phịt hiỷn phÝa bến kia ệđêng cã mét
ệịm ệềng. Linh cờm nghÒ nghiỷp khiạn ềng
cờm thÊy hừnh nhđ cã chuyỷn gừ khềng ữn.
“Chớc mừnh lỰi chđa thĨ vỊ nhộ ệđĩc răi!”
-thịm tỏ thẵm nghỵ... Răi ềng quyạt ệỡnh sang
ệđêng, tắi tẺn chẫ ệịm ệềng ệang tô tẺp.
Thừ ra ềng M. chự cỏa hộng ệiỷn thoỰi di ệéng
võa bỡ mÊt trém. Hộng xãm vộ ngđêi ệi ệđêng
ệang xóm lỰi bộn tịn, hái han. Thịm tỏ
Sếlềccềc bđắc vộo cỏa hộng vộ hái ềng M.:
- Cã chuyỷn gừ vẺy? Tềi lộ thịm tỏ Sếlềccềc
ệẹy... Liỷu tềi cã thÓ gióp gừ cho ềng khềng?
ngộy tềi vÉn thÊy ềng ệi bé qua ệẹy... Khềng
ngê ềng chÝnh lộ ngđêi mộ tềi luền ngđìng


mé.


- Hình nh ông vừa bị mất trộm?


- Võng... My cỏi máy điện thoại đắt tiền nhất
vừa bị mất.


- ChiÒu nay, cờ ba nhẹn viến bịn hộng ệÒu vÒ
sắm, chử cưn tềi ẻ lỰi cỏa hộng. Khoờng 7h30,
tềi ệang bớt ệẵu thu dản ệÓ ệãng cỏa thừ cã
mét khịch hộng bđắc vộo. Anh ta nãi muèn
mua loỰi mịy X. Lóc Êy, nhọng chiạc mịy loỰi
ệã ệang ẻ trến giị cao nến tềi phời bđắc lến
ghạ ệÓ lÊy. Võa bđắc xuèng, cưn chđa kỡp ệđa


mịy cho anh ta thừ tềi bẫng thÊy chãng mẳt.
Tềi ngăi véi vộo ghạ, cè nhê anh ta rãt gióp
cèc nđắc... Răi tềi mỷt lờ ệi... Ngđêi khịch thừ
hừnh nhđ vÉn chẽm chó ngớm nghÝa mÊy
chiạc mịy. Mét lịt sau, anh ta bờo tềi: “Thềi,
chó ệang mỷt thừ cụ nghử ệi. Mai chịu qua
xem tiạp”. Anh ta ệi răi, tềi còng dẵn dẵn tửnh
tịo trẻ lỰi... Răi tềi phịt hiỷn mÊy chiạc mịy
ệớt tiÒn trong tự kÝnh ệở bỡ mÊt.


- Loại máy bị mất có trùng với loại anh ta hái


AI LÀ KẺ



KHẢ NGHI?


Ngun Tn Anh



</div>
<span class='text_page_counter'>(44)</span><div class='page_container' data-page=44>

43


mua kh«ng?


- Không! Máy bị mất là loại rất đắt, tôi bày
trong ngăn kính của bàn quầy.


kh«ng?


- Anh ta cịn trẻ, tầm tuổi nhð mấy nhân viên
của tơi. Hình nhð anh ta có vẻ gì đó quen
quen nhðng tơi khơng nhớ ra.


- Anh ta ệéi mị bờo hiĨm, ệeo kÝnh nến tềi


khềng nhừn râ mẳt. Vắi lỰi lóc ệã tềi ệang dản
dứp, chuÈn bỡ ệãng cỏa nến còng khềng tẺp
trung chó ý lớm. Tềi lỰi phời ệi từm ghạ, răi kế
ghạ bđắc lến... Lóc xuèng thừ chãng mẳt, hoa
mớt... Cã biạt gừ nọa ệẹu... Mừnh nhiỊu tuữi răi,
khềng thĨ minh mÉn nhđ hăi trĨ ệđĩc, thịm tỏ
nhử!


- §óng vËy... Nhng bây giờ bình tĩnh rồi, ông
có nghi ngờ ai không?


- Tôi cha nghĩ tới khả năng này.


- Ti th lỰi nghi mét ai ệã trong sè nhẹn viến
cựa ềng. Tềi cẵn gẳp tõng ngđêi ệÓ xem mừnh
nghi ngê cã ệóng khềng.


Ngay sau ệã, ềng M. gải ệiỷn cho 3 nhẹn viến
cựa mừnh vộ thịm tỏ Sếlềccềc ệở nãi chuyỷn
qua ệiỷn thoỰi vắi tõng ngđêi mét. ậẵu tiến lộ
anh H.


- Tôi là thám tử Sêlôccôc đây! Cửa hàng nơi
anh làm việc vừa bị mất trộm. Anh có thể cho
biết, trong khoảng từ 7 đến 8 giờ tối nay, anh


đã làm gì, ở đâu?


- Thða thám tử! Hơm nay tơi xin phép ông chủ
về sớm để đða mẹ đi siêu thị. Hai mẹ con đã


mua 1 cái nồi cơm điện. Tôi cịn giữ hóa đơn
và phiếu bảo hành đây ạ.


TiÕp theo lµ anh N.


- Tơi là thám tử Sêlơccơc. Tơi cần biết anh đã
làm gì tối nay, trong quãng thời gian t 7 n
8 gi?


- Dạ tha... Tôi chỉ ở nhà xem TV thôi. Tôi mệt
nên xin về sớm mà.


- Anh xem chđểng trừnh gừ vẺy?


- Tềi xem phim trinh thịm trến VTV3. Tuẵn
nộo còng vẺy, cụ ệạn ngộy cã bé phim ệã lộ
tềi phời xem bỪng ệđĩc.


Ci cïng lµ anh V.


- Tơi là thám tử Sêlôccôc. Tôi muốn biết trong
khoảng thời gian từ 7 đến 8 giờ tối nay, anh đã
làm gì, đi đâu?


- Tềi vộ bỰn gịi ệạn rỰp xem phim. Chóng tềi
xem tõ 7 rđìi tắi 9 giê. Tềi võa vỊ nhộ ệđĩc
mét lóc thừ ềng gải ệÊy...


- Anh xem ë r¹p nào?



- Rạp Cầu Vồng ạ. Tôi vẫn còn giữ cuống vé
đây.


Sau khi hỏi chuyn c 3 ngời, thm t nói vắi
ềng M.:


- Cã mét ngđêi khiạn tềi rÊt nghi ngê. Cã lỳ
chóng ta cẵn ệạn nhộ anh ta ngay.


* Theo các bạn, thám tử đã nghi ai? Vì sao?


ậu bé đã xé vội tờ giấy có hình con nhện.
Điều này khơng phải là tình cờ, bởi nói đến


nhỷn lộ chóng ta liến tđẻng ngay ệạn mỰng
nhỷn - net. CẺu bĐ ệở cã ý bịo cho mải ngđêi
vÒ bộ chự quịn trư chểi ệiỷn tỏ.


RÊt nhiÒu bỰn gỏi bội vộ hẵu hạt ệÒu cã suy
ệoịn nhđ vẺy. Phẵn thđẻng sỳ ệđĩc gỏi tắi: Bỉi
Chẹu Anh, 7A, THCS PhÊn MƠ I, Phó Lđểng,


Thịi Nguyến;Ngun Dđểng Hoộng Anh, 7C,
THCS Vẽn Lang, Viỷt Trừ, Phó Thả; Ngun
Trung Kiến, 6A, THCS Vỵnh Tđêng, Vỵnh
Tđêng,Vỵnh Phóc;Bỉi Thỡ Anh Thể, 6C, THCS
Lý NhẺt Quang, ậề Lđểng; Cao Khớc Tẹn, 7A,
THCS Cao Xuẹn Huy, DiƠn Chẹu, Nghỷ An.


Th¸m tư Sêlôccôc



</div>
<span class='text_page_counter'>(45)</span><div class='page_container' data-page=45></div>
<span class='text_page_counter'>(46)</span><div class='page_container' data-page=46>

45



UNIT 9.

TRANSFER OF HEAT



Vị Kim Thđy



Q1.

B. by radiation only


Q2.

C. vacuum.



NhẺn xĐt.

Bội trớc nghiỷm khềng khã nhđng


Ýt bỰn giời ệóng. Cịc bỰn hởy cè gớng ẻ nhọng


bội sau. Cịc bỰn ệđĩc thđẻng kừ nộy:

NguyÔn


NhẺt nh

, 7C1, THCS Quờng Ngảc, Quờng



Xđểng,

Thanh Hãa

;

TỰ Họu Tiạn Thộnh

,


7A1, THCS Cao Xuẹn Huy, DiÔn Chẹu,

Nghỷ


An

;

NguyÔn Lế Hđểng Ly

, 8C, THCS Nam


Cao, Lý Nhẹn,

Hộ Nam

;

Tèng BÝch Ngảc

, 9A,


THCS Tẹn Bừnh, TX. Tam ậiỷp,

Ninh Bừnh

.



§inh thu


(TTT2 sè 134)


Question 3.

An unlit



match is held near


to an extremely hot


bunsen flame. The


match does not get


hot enough to light



because



A. the flame is not hot enough


B. air is a bad conductor of heat


C. the match head reflects radiation



D. the flame does not radiate any heat


sideways



E. a match can only be lit by striking it on a


rough surface



Question 4.

How is heat transferred through


the walls of a steel radiator?



A. conduction only


B. convection only


C. radiation only



D. conduction and convection



E. convection and radiation



Question 5.

In the process of convection,


heat energy is transferred



A. by electromagnetic radiation



B. because of temperature differences in a


solid




C. because of density differences in a fluid


D. by the vibration of molecules about a


mean position



E. by the diffusion of molecules through a


fluid



Question 6.

Which of the following will be the


best absorber of infra-red radiation?



A. dark animal fur


B. shiny metal tray


C. white plastic bag


D. window glass


E. writing paper


Physics Terms



radiator

lư sđẻi



sideways

ra xung quanh



fluid

chÊt



vibration

dao ng



diffusion

khuếch tán



absorber

hấp thụ




plastic bag

túi ni lông



glass

kính



electromagnetic radiation

bức xạ điện từ


Answer.



</div>
<span class='text_page_counter'>(47)</span><div class='page_container' data-page=47>

46


Bi ton.Gii phng trnh nghim nguyn dng


. (1)


Lời giải.Cách 1.


Ta có (1) 7(x2 y2) 25(x y)
x(7x 25) y(25 7y). (2)


+) NÕu 7x 25 0 th× tõ (2) suy ra 25 7y 0
y {1; 2; 3} (v× y +).


Tõ ệã ta từm ệđĩc bé sè nguyến dđểng
(x; y) (4; 3) tháa mởn.


+) NÕu 7x 25 0 th× x {1; 2; 3}.


Ta ệđĩc bé sè nguyến dđểng (x; y) (3; 4) tháa
mởn.


VẺy nghiỷm nguyến dđểng cựa phđểng trừnh lộ
(x; y) (3; 4), (4; 3).



C¸ch 2.Tõ (1) suy ra
7x2 7y2 25x 25y 0


(14x 25)2 (14y 25)2 1250. (3)
Tõ (3) suy ra (14x 25)2 1250.


Mộ x +nến 35 14x 25 35 x 4.
Tđểng tù y 4.


Tõ ệã từm ệđĩc nghiỷm nguyến dđểng cựa phđểng
trừnh lộ (x; y) (3; 4), (4; 3).


C¸ch 3.Đặt (t *).


Vì (x y)2 0 nên x2 y2 2xy
2(x2 y2) (x y)2.


Suy ra 50t 2(x2 y2) (x y)2 49t2.
Do đó t 1.


VËy (1) trë thµnh


x2 7x 12 0 (x 3)(x 4) 0
x {3; 4}.


Tõ ệã từm ệđĩc (x; y) (3; 4), (4; 3).


C¸ch 4.Ta cã



(1) 7x2 25x (7y2 25y) 0. (4)


Coi (4) lộ phđểng trừnh bẺc hai Èn x, vắi y lộ tham sè.
Ta cã 252 28(7y2 25y) 0


(14y 25)2 1250.
Tđểng tù cịch 2 suy ra x 4.
Tõ ệã từm ệđĩc (x; y) (3; 4), (4; 3).


Cịch 5.Tđểng tù cịch 3 ta cã
2(x2 y2) (x y)2.


Do đó từ (1) suy ra
x y 7.


Mặt khác, từ (1) ta thÊy x y 7 nªn x y 7.
Suy ra x2 y2 25.


Đến đây ta giải tiếp nh cách 3.


Qua cịc cịch giời bội toịn trến hy vảng cã thĨ
gióp cịc em hảc sinh thÊy ệđĩc sù phong phó vỊ
cịch giời, cịng nhđ phđểng phịp giời phđểng
trừnh nghiỷm nguyến.


2 2
7 x y 1
50 <sub>2(x</sub> <sub>y )</sub> x y


2 2



x y 7 <sub>x y 7</sub> <sub>y 7 x</sub>
xy 12 x(7 x) 12
x y 25


2 2
x y 7t
x y 25t
2 2


7 x y
25 x y


GIẢI MỘT BÀI TỐN SỐ HỌC



ậộo Huy Trđêng


(P. Hiỷu trđẻng THCS LẺp ThỰch, LẺp ThỰch, Vỵnh Phóc)


</div>
<span class='text_page_counter'>(48)</span><div class='page_container' data-page=48>

47



Lời giải.(Theo lời giải của bạn Nguyễn Thành Vinh, 8H, THCS Lê Quý Đôn, Cầu Giấy, Hà Nội)
Ta có


áp dụng bất đẳng thức (với x, y 0) ta có


Theo bất đẳng thức AM-GM ta có


Tđểng tù
Suy ra



Mặt khác Do đó


Suy ra DÊu ệỬng thục xờy ra khi a b c 1. VẺy khi a b c 1.
BỰnNguyÔn Thộnh Vinhlộ ngđêi ệẽng quang trong trẺn ệÊu nộy.


Cịc bỰn sau cịng cã lêi giời ệóng: Ngun Thỡ Thanh Hđểng, NguyÔn Thỡ Hiến, 9A, THCS Yến Phong,
Yến Phong, Bớc Ninh;Trẵn Thu HuyÒn, 9A, THCS Tam Dđểng, Tam Dđểng, Vỵnh Phóc;Lế Bờo Anh,
9A, THCS Phong Chẹu, TX. Phó Thả; Ngun MỰnh Mđêi, 9A, THCS Thanh Thựy, Thanh Thựy, Phó
Thả;Ngun Phỉng Thịi Cđêng, 9B, THCS Hưa Hiạu II, TX. Thịi Hưa, Nghỷ An;NguyÔn Trẵn Khang,
8B, THCS NguyÔn Du, TP. Hộ Tỵnh, Hộ Tỵnh;NguyÔn Vẽn Cao, NguyÔn TuÊn Anh, 8A, THCS Ngun
Thđĩng HiỊn, ụng Hưa, Hộ Néi.


Ngun Ngäc H©n


1
MaxP


4
1


4P 1 P
4


1 1 1 abc 1 b


ab a 1 bc b 1 ca c 1 ab a abc bc b 1 abc bc b
bc 1 b bc <sub>b 1 1.</sub>


b 1 bc bc b 1 1 bc b bc b 1



abc 1 abc 1


1 1 1


4P .


ab a 1 bc b 1 ca c 1


1 1 1


2P


2 ab 2 a 2 2 bc 2 b 2 2 ca 2 c 2


1 1 <sub>;</sub> 1 1 <sub>.</sub>


2b c 3 2 bc 2 b 2 2c a 3 2 ca 2 c 2


1 1


2a b 3 (a b) (a 1) 2 2 ab 2 a 2 .
2a b 3 2 ab 2 a 2


1 1 1 1 1 1


4P


2a b 3 2b c 3 2b c 3 2c a 3 2c a 3 2a b 3



1 1 1


2P .


2a b 3 2b c 3 2c a 3
1 1 4
x y x y


4 4 4


4P .


(2a b 3) (2b c 3) (2b c 3) (2c a 3) (2c a 3) (2a b 3)


Ngđêi thịch ệÊu:Dđểng ậục Lẹm, GV THPT chuyến ậỰi hảc Sđ phỰm Hộ Néi.


Bài toán thách đấu:Cho a, b, c Chứng minh rằng


XuÊt xø: S¸ng t¸c.


Thêi hỰn:Trđắc ngộy 08.09.2014 theo dÊu bđu ệiỷn.


3 3 3


2 2 2


a b b c c a <sub>4 1</sub> 3abc <sub>.</sub>
a b b c c a
ab c bc a ca b



TRẬN ĐẤU THỨ MỘT TRĂM MƯỜI TÁM



</div>
<span class='text_page_counter'>(49)</span><div class='page_container' data-page=49>

48



MỞ RỘNG



HAI ĐỀ TỐN HAY


TrÇn Quang Hïng(GV. THPT chuyên Đại học KHTN)


Trên TTT2 số 92 năm 2010 mục giải toán qua th
có bài toán hay nh sau của thầy Nguyễn Minh Hà:


Bi ton 1. Cho tam gic ABC vuềng tỰi A. Hừnh
vuềng MNPQ cã M thuéc cỰnh AB, N thuéc cỰnh
AC vộ P, Q thuéc cỰnh BC. Giờ sỏ BN cớt MQ tỰi
E, CM cớt NP tỰi F. Chụng minh rỪng AE AF vộ
Lêi giời bội toịn trến ệở cã trến TTT2 sè 94 nẽm
2010. Phẵn chụng minh hai gãc bỪng nhau ệđĩc
mẻ réng cho hừnh chọ nhẺt cựa cỉng tịc giờ vộ cã
trến TTT2 sè 127 nẽm 2013 trong mơc thịch ệÊu
nhđ sau:


Bài tốn 2. Cho tam giác ABC vng tại A. Hình
chữ nhật MNPQ thay đổi thỏa mãn M thuộc cạnh
AB, N thuộc cạnh AC và P, Q thuộc cạnh BC. BN
cắt MQ tại K, CM cắt NP tại L, BN cắt CM tại X,
QN cắt PM tại Y.


a) Chøng minh r»ng



b) Chứng minh rằng XY luôn đi qua một điểm cố định.
Lời giải bài tốn trên đã có trên TTT2 số 129 năm
2013. Bài báo này sẽ trình bày một số mở rộng
cho hai bài toán trên. Một cách tự nhiên chúng ta
suy nghĩ rằng liệu bài tốn 1 có thể có cách phát
biểu nào cho tam giác bất kì. Ta đi đến mở rộng
đầu tiên nhð sau:


Bội toịn 3. Cho tam giịc ABC nhản, khềng cẹn.
Dùng hừnh chọ nhẺt MNPQ sao cho M thuéc cỰnh
AB, N thuéc cỰnh AC vộ P, Q thuéc cỰnh BC (vắi
P nỪm giọa Q, C) tháa mởn ậđêng
thỬng qua A vuềng gãc AB cớt NP tỰi K. ậđêng
thỬng qua A vuềng gãc AC cớt MQ tỰi L. CL cớt NP
tỰi E. BK cớt MQ tỰi F. Chụng minh rỪng AE AF.


Lêi gi¶i. Gọi AD là phân giác của ABC. Giả sử
AL, AK cắt BC thứ tự tại S, T. Ta sẽ chøng minh
r»ng DE // SL.


Thật vậy, vì CPN CAS, CQN CAD
nên áp dụng định lí Thales, ta có


Từ đó DE // SL.


Mộ SL AC nến DE AC tỰi G.
Tđểng tù DF AB tỰi H.


Ta thÊy cịc ệiÓm A, M, N, K, L thuéc ệđêng trưn
ệđêng kÝnh KM, LN nến MNKL lộ hừnh chọ nhẺt.


VẺy còng tõ ệỡnh lÝ Thales ta cã


Chú ý rằng AD là phân giác góc A và G, H là hình
chiếu của D lên CA, AB nên DG DH. T ú ta cú
DE DF.


Mà nên


ADE ADF (c.g.c).
Do đó AE AF (đpcm).


NhẺn xĐt. Trong chụng minh trến ta suy ra
Nạu trong bội toịn 1, ta coi MQ, NP
lộ cịc tiạp tuyạn cựa ệđêng trưn ngoỰi tiạp tam
giịc vuềng AMN thừ ta sỳ cã mét hđắng mẻ réng
khịc nhđ sau.


Bội toịn 4. Cho tam giịc ABC néi tiạp ệđêng trưn
(O). Tiạp tuyạn tỰi B, C cựa ệđêng trưn (O) cớt
nhau tỰi T. Gải M, N lẵn lđĩt lộ cịc ệiÓm thuéc tia
BT, CT sao cho BM CN BC. ậđêng thỬng MN
cớt CA, AB theo thụ tù tỰi E, F. BE cớt CT tỰi P, CF
cớt BT tỰi Q.


Chøng minh r»ng AP AQ.


Lời giải.Gọi AD là phân giác của ABC.


Vỡ B, C đối xứng nhau qua OT và BM CN nên
M, N đối xứng qua OT. Suy ra BC // MN.



Ta có


Mà nên ABC MFB.


T đó, với chú ý FM // BC nên theo định lí Thales ABC BFM


o


180 ABC CAB ACB.
o


FBM 180 ABC CBM
EAB FAC.


o BAC


EDA FDA ( 90 )
2
DE SL TK DF .
DG SA TA DH


CE CP CP CN CA CD CD<sub>.</sub> <sub>.</sub> <sub>.</sub>
CL CQ CN CQ CS CA CS


BAC
MNQ .


</div>
<span class='text_page_counter'>(50)</span><div class='page_container' data-page=50>

49




ta cã


Suy ra QD // BF. Tđểng tù PD // CE.


Tõ ệã theo ệỡnh lÝ Thales vộ tÝnh chÊt ệđêng phẹn
giịc ta cã


VËy DP DQ. (1)


V× nªn


Tõ (1), (2) suy ra ADQ ADP (c.g.c).
VËy AQ AP (®pcm).


NhËn xÐt.Trong chøng minh trªn ta thÊy


Hđắng mẻ réng cựa bội toịn 3 cịng
cã thĨ ịp dơng tiạp ệđĩc cho bội toịn 2.


Ta có bài toán nh sau:


Bi ton 5. Cho tam giịc ABC. Dùng hừnh chọ nhẺt MNPQ sao cho M, N lẵn lđĩt thuéc AB, AC vộ P, Q
thuéc BC. ậđêng thỬng qua A vuềng gãc AB cớt NP tỰi K. ậđêng thỬng qua A vuềng gãc AC cớt MQ tỰi
L. CL cớt NP tỰi E, BK cớt MQ tỰi F.


a) Chøng minh r»ng


b) Giả sử LP cắt QK tại X, BK cắt CL tại Y. Chứng minh rằng XY luôn đi qua một điểm cố định khi hình
chữ nhật MNPQ thay đổi.



Lêi giời.a) Giờ sỏ AL, AK cớt BC lẵn lđĩt tỰi S, T. Gải I, J lẵn lđĩt lộ hừnh chiạu cựa C, B lến AE, AF.


Vì BMQ BAT, CNP CSA nên


T ú ABJ ACI nên


b) Gải giao ệiÓm cựa XY vắi LK, BC lẵn lđĩt lộ Z, H.


Vừ tụ giịc LKPQ lộ hừnh chọ nhẺt, X lộ giao ệiÓm hai ệđêng chĐo nến theo ệỡnh lÝ Thales ta cã


Suy ra AH // NP.


Mà NP BC nên AH BC.
Do đó H cố định.


Vậy XY đi qua H cố định (đpcm).


HP LZ <sub>LY LK MN AN.</sub>
HC HC YC BC BC AC


BAF CAE.


BJ BJ AK AL BF AK LE BQ AT LK BQ.AT BQ AB AC AT<sub>.</sub> <sub>.</sub> <sub>.</sub> <sub>.</sub> <sub>.</sub> <sub>.</sub> <sub>.</sub> <sub>.</sub> <sub>.</sub>
CI AK AL CI FK AL EC LK AS PC AS.PC AB AC PC AS


MQ AB AS AT AB<sub>.</sub> <sub>.</sub> <sub>.</sub> <sub>.</sub>
AT AC NP AS AC


EAC FAB.
PAB QAC.



ADQ ADP. (2)
xDQ xAB xAC xDP


DQ DQ BF CE CD AB BC CD AB<sub>.</sub> <sub>.</sub> <sub>.</sub> <sub>.</sub> <sub>.</sub> <sub>1</sub>
DP BF CE DP BC AC BD BD AC


</div>
<span class='text_page_counter'>(51)</span><div class='page_container' data-page=51>

50


Câu 6.

Bạn hãy điền các chữ cái A, B, C, D,


E, F vào những tam giác nhỏ, sao cho ở mỗi


lục giác (gồm 6 tam giác nhỏ liền kề hợp


thành) đều có đủ 6 chữ cái trên.



Cẹu 7.

BỰn hởy kĨ ệđêng thỬng ngang, dảc


tỉy ý bớt ệẵu tõ chọ T ệạn ầ ệĨ ghĐp tuẵn tù


ệđĩc cơm tõ

tỰp chÝ toịn tuữi thể

.



ậđêng kĨ khềng chăng chĐo nhau vộ phời


qua hạt 56 ề vuềng.



Cẹu 8.

BỰn hởy dỉng cịc dÊu , , , :, ngoẳc


ệển ệÓ chen vộo giọa nhọng dởy sè dđắi ệẹy,


sao cho ệđĩc kạt quờ bỪng 2014 nhĐ!



1 1 1 1 1 1 1 1 1 1 1 1 1 1 2014


2 2 2 2 2 2 2 2 2 2 2 2 2 2014


3 3 3 3 3 3 3 3 3 3 3 3 3 3 3 3 2014



</div>
<span class='text_page_counter'>(52)</span><div class='page_container' data-page=52>

51




KÌ 13


BỰn hởy thay mẫi chọ
cịi bẻi mét chọ sè
sao cho ệđĩc phĐp
tÝnh ệóng, biạt rỪng
cịc chọ cịi khịc
nhau biÓu thỡ cịc chọ
sè khc nhau. Lời gii
cn ghi rõ lp lun.


Đánh số 5 cột từ phải qua trái.


Gi s khi thực hin cịc phĐp céng theo cét, sè
nhắ ẻ cịc cét 1, 2, 3 lẵn lđĩt lộ x, y, z.


Ta thÊy x, y, z 3.


XÐt cét 4 vµ cét 5, suy ra S z 8T A.
V× S z 9 3 12 nên 8T A 12.
Suy ra T 1 và S z 8 A.


XĐt trđêng hĩp S 8, z 0,A 0.


(Cịc trđêng hĩp khịc bỰn ệảc tù giời, ệÒu dÉn
tắi về nghiỷm)


Tõ cét 1 ta cã 2D L 1 8 10x hay
2D L 7 10x. (1)


V× S 8 nªn 2D L 18 7 25 nªn tõ (1) suy


ra x {0; 1}.


TH1.XÐt x 0. Từ (1) 2D L 7.
Vì A 0 và T 1 nªn (D; L) (2; 3).


Thay vộo cét 2 ta ệđĩc E 2: loỰi (vừ D 2).


TH2.XÐt x 1. Tõ (1) 2D L 17.
V× T 1, S 8 nªn


(D; L) (4; 9), (5; 7), (6; 5), (7; 3).
+ XÐt (D; L) (4; 9).


Tõ cét 2 suy ra E 7 vµ y 3.


Tõ cét 3 suy ra 2O 3 9 nến O 3.
Ta ệđĩc


+ XÐt (D; L) (5; 7).


Tõ cét 2 suy ra E 9 vµ y 2.
Tõ cét 3 suy ra 2O 2 7: lo¹i.
+ XÐt (D; L) (6; 5).


Tõ cột 2 suy ra E 1: loại vì T 1.
+ XÐt (D; L) (7; 3).


Tõ cét 2 suy ra E 3: loại vì L 3.


Nhn xt. Bội nộy giời khị dội vộ chử dÉn ệạn


mét ệịp sè nhđ trến. Cịc bỰn sau ệở lẺp luẺn ệÓ
từm ệóng ệịp sè: Ngun Thộnh Cđểng, 9C,
THCS Tẹn Bừnh, TP. Hời Dđểng, Hời Dđểng;


Hoộng Thạ Sển, NguyÔn Phđểng Thờo Vy, 7A1,
THCS Hăng Bộng, Hăng Bộng, Hời Phưng.


Hoộng nguyến linh
Trđểng Cềng Thộnh(Sđu tẵm)


</div>
<span class='text_page_counter'>(53)</span><div class='page_container' data-page=53>

52


Bµi 1. TÝnh


;


Bµi 2.Rót gọn
;


Bài 3.Tìm x biết


a) ; b) ;


c) ; d) ;


Bài 4.Tìm a, b, c biÕt:


a) 2a 3b 4c vµ a b c 28;
b) ; vµ a b c 3;


Bµi 5.



a) Tỉ số giữa số học sinh nữ và sè häc sinh nam
cđa mét líp häc lµ 0,85. TÝnh số học sinh nam và
số học sinh nữ biết số häc sinh cđa líp 7A lµ 37
em.


b) Tử sè giọa sè cẹy trăng ệđĩc cựa lắp 7A vộ lắp
7B lộ 0,75. Sè cẹy trăng ệđĩc cựa lắp 7A Ýt hển sè
cẹy trăng ệđĩc cựa lắp 7B lộ 10 cẹy. Hái sè cẹy
trăng ệđĩc cựa mẫi lắp lộ bao nhiếu?


c) Chøng minh r»ng


1 2 3 ... 9 12 5 5.
b c


2 3
a b


2 3


x 5 3
2 4 .4
3
1 8
x
5 27
2
1 9
2x


3 25
1 7 <sub>: x</sub> 1 14


15 12 2 15


0 2


1998 1
D 15 : 2.


1999 2
11 2
3 2
2 .9
C
6 .116
2
1


B 0,9 ( 0,5) .
4


9 1 5 3 5 3 9
A 5 4 1


4 27 18 7 3 5 10


ĐỀ KIỂM TRA CHƯƠNG I ĐẠI SỐ 7



Thời gian làm bài:45 phút (không kể thời gian giao đề)



MÃ ĐỀ: RDKTH011



1(137+138).Compare and 3, where


A 1 22014 32013 42012 ... 20142 2015,
andB 1 22013 32012 42011 ... 20132 2014.


2(137+138). Given that


x,y,zare integers such
that (x3 y3 z3) 27,
prove that either x,y,z


are all divisible by 3 or
the sum of two numbers
among them are divisible
by 9.


3(137+138). Solve the
equation


4(137+138). Let a, b, and c be positive real
numbers such that


Prove that


5(137+138).Given the set P {red, green, back, white}.
Determine if each of the followings is a partition of P.
a)P<sub>1</sub> [{red}, {green, black}]



b)P<sub>2</sub> [{white, black, red, green}]
c)P<sub>3</sub> [ , {red, green}, {black, white}].


6(137+138). Let O be a point in the equilateral
triangle ABC. Given that the area of the shaded
region is half the area of the triangle. Prove that
the point O lies on one of the medians of the
triangle.


1 1 1 <sub>3.</sub>
6a 1 6b 1 6 1 7c


1 1 1 <sub>1.</sub>
2 1 2a b 1 2 1c


2 x 1 <sub>2</sub> <sub>x</sub> <sub>1</sub> <sub>x</sub> <sub>x</sub> <sub>1.</sub>


x x
A


B


Translated by Nam Vị Thµnh



</div>
<span class='text_page_counter'>(54)</span><div class='page_container' data-page=54>

53


Bài 10NS.Viết số 20132014 thành tổng các số tự
nhiên, kí hiệu S là tổng các lũy thừa bậc 7 của các
số tự nhiên đó. Tìm s d khi chia S cho 42.



Trần Văn Hng


(GV. THCS Yên Thanh, Can Lộc, Hà Tĩnh)
Bài 11NS.Cho các số thực a, b tháa m·n a 1,
b 1, a b 3. Tìm giá trị nhỏ nhất của biểu thức:


Lại quang thä


(Chuyến viến phưng Giịo dôc & ậộo tỰo
Tam Dđểng, Vỵnh Phóc)
Bội 12NS. Cho tam giịc ABC (AB AC), AD lộ
ệđêng phẹn giịc. Vỳ DE vuềng gãc vắi AB tỰi E,
DF vuềng gãc vắi AC tỰi F. ậđêng thỬng vuềng
gãc vắi BC tỰi D cớt EF tỰi K. CK cớt AB tỰi M, BK
cớt AC tỰi N. Chụng minh rỪng MN // BC.


nguyễn đức tấn


(TP. Hå ChÝ Minh)


2 2 1 1 1
P a b a 2b .


a b a b


Bội 4NS. Bữ ệÒ: Nạu a, b, n lộ cịc sè tù nhiến
tháa mởn n2 a.b vộ (a, b) 1 thừ a, b ệÒu lộ sè
chÝnh phđểng. (BỰn ệảc tù chụng minh)


Ta cã 3x2 x 4y2 y


(x y)(4x 4y 1) x2.


Gäi d (x y, 4x 4y 1) th× d 1.


Do ệã x y vộ 4x 4y 1 ệÒu lộ sè chÝnh phđểng.
Suy ra A 2xy 4(x y)3 x2 y2


(x y)2 4(x y)3 (x y)2(4x 4y 1) lộ sè
chÝnh phđểng.


NhẺn xĐt.Cịc bỰn sau cã lêi giời tèt: NguyÔn Thỡ
Thanh Hđểng, 9A, THCS Yến Phong, Yến Phong,


Bớc Ninh; Lế NguyÔn Quúnh Trang, 7C, THCS
Vẽn Lang, TP. Viỷt Trừ; NguyÔn Thỉy Dđểng, 6A3,
THCS Lẹm Thao, Lẹm Thao, Phó Thả; Ngề Lế
Phđểng Trinh, 9E, THCS Lđểng Thạ Vinh, TP. Tuy
Hưa, Phó Yến; Ngun Thỡ Khịnh Hun, 8A3,
THCS Tõ Sển, Long Biến, Hộ Néi.


Bội 5NS.Ta thÊy x 2 vộ x 1 ệÒu lộ nghiỷm
cựa phđểng trừnh ệở cho.


Ta cã (x 1)6 (x 2)4 x2 3x 3


(x 1)6 (x 2)4 (x 1)2 (x 2).
* NÕu x 2 th× (x 1)6 (x 2)4 (x 1)2
(x 2).


* NÕu 2 x 1 th× (x 1)6 (x 2)4 (x 1)2


(x 2).


* Nạu x 1 thừ x 1 0. ậẳt x 1 y 0
Ta ệđĩc phđểng trừnh y6 (y 1)4 y2 y 1


y6 y4 4y3 5y2 3y 0 (Phđểng trừnh nộy
về nghiỷm vừ y 0).


VẺy phđểng trừnh ệở cho cã hai nghiỷm x 2;
x 1.


NhẺn xĐt.Cịc bỰn sau cã lêi giời tèt: NguyÔn Thỡ
Thanh Hđểng, 9A, THCS Yến Phong, Yến Phong,


Bớc Ninh; Lế NguyÔn Quúnh Trang, 7C, THCS
Vẽn Lang, TP. Viỷt Trừ, Phó Thả;Ngề Lế Phđểng
Trinh, 9E, THCS Lđểng Thạ Vinh, TP. Tuy Hưa,


Phó Yến; Ngun Thỡ Thờo Phđểng, 8A, THCS
ậục Lý, Lý Nhẹn, Hộ Nam;NguyÔn Thỡ HỪng, 8B,
THCS Lý NhẺt Quang, ậề Lđểng, Nghỷ An.


(Xem tiÕp trang 59)


</div>
<span class='text_page_counter'>(55)</span><div class='page_container' data-page=55>

54



ASIA PACIFIC MATHEMATICAL


OLYMPIAD 2014



2 hours (150 marks)




1. The figure show a regular octagon. Find the
value of angle SRT in degrees, where R is the
centre of the star.


2.Which of the following statements is correct?
(1) (2)


(3) (4)
(5)


3.Find the smallest positive integer with sum of
digits equal to 29.


4. Abel and Jim run a race together on a 300
metres track. They start simultaneously at the
same point. Abel runs at a constant speed of
5 m/s, white Jim runs at a constant speed of
4.2 m/s. How many full laps has Abel run before
he is able to overtake Jim for the first time?


5. Only one of the following four number is a
perfect square. Which one is it?


(1) 76186 (2) 750235
(3) 921438 (4) 2660161


6.As shown in the diagram, ABC is an isosceles
right-angle triangle with AB 28 cm. BC is the
diameter of the semi-circle and point D is the



midpoint of arc BC. Find the area of the shaded
reigion in cm2.


(Take to be ).


7. Three chess pieces, each of the colour red,
black and white, are to be placed on a 7 7
chessboard. If any two of the three pieces cannot
be placed in the same row or the same column,
how many ways are there to place the three
chess pieces?


8.There are 6 bags containing 18, 19, 21, 23, 25
and 34 balls respectively. One bag contains only
red balls while the other five bags contain only
blue balls. Jason takes three bags and Jamie
takes two bags. The remaining bag contains red
balls. It is known that now Jason has twice as
many blue balls as Jamie does. Find the number
of red balls in the remaining bag.


9.A water tank can be filled by using tap A for 8
hours followed by tap B for 15 hours. The same
tank can also be filled by using tap A for 5 hours
followed by tap B for 24 hours. How long will it
take (in hours) to fill the tank by using tap A only?


10.A square piece of paper is folded along the
diagonals twice, as shown in the diagrams below.


Then a cut is made along the dotted line.


22
7


7 3 15
9 4 19


3 7 15
4 9 19
7 15 3


9 19 4


3 15 7
4 9 9
15 7 3


</div>
<span class='text_page_counter'>(56)</span><div class='page_container' data-page=56>

55



When the paper is unfolded, which one of the following diagrams showns how the paper appears?


11.Find the value of 20142013 20132014 20132013 20142014.


12.Alan, Ben, Chris and Daniel took part in a science quiz. The quiz consisted of 5 True of False
questions. The table below shown the answers given by each of them and their scores. Each correct
answer was given 1 point.


How many points did Daniel score?



13.A computer program generated all five-letter code words that can be formed by using letters A, B,
C, D, E, F, G, X, Y, Z (repetition is allowed). When all these words are sorted according to the
alphabetical order, the following list is obtained: AAAAA, AAAAB, ... , AAAAZ, AAABA, AAABB, ... ,
ZZZZY, ZZZZZ. Find the number of code words between CZYGB and XEFDA, not including these two
code words.


14.Find the largest 3-digit interger N such that when N is divided by 3, 7, 11, the remainders are 1, 3
and 8 respectively.


15.Find the value of


K× sau đăng tiếp
1 5 11 89


10 ... .


</div>
<span class='text_page_counter'>(57)</span><div class='page_container' data-page=57>

56



Ngy nay, viỷc giời phđểng trừnh bẺc hai, bẺc ba
vộ bẺc bèn ệở khềng cưn xa lỰ vắi nhiÒu bỰn hảc
sinh. Tuy vẺy, khềng phời ai còng biạt nguăn gèc
ra ệêi cựa nhọng phđểng trừnh nộy. Chóng ta cỉng
từm hiĨu.


Khoờng nẽm 400 trđắc Cềng nguyến, ngđêi
Babylon ệở biạt giời phđểng trừnh bẺc hai dỉ hả
khềng cã khịi niỷm phđểng trừnh. Hả ệở phịt triÓn
ệđĩc mét thuẺt toịn ệĨ giời quyạt vÊn ệỊ mộ theo
thuẺt ngọ hiỷn ệỰi lộ viỷc giời phđểng trừnh bẺc
hai. Hả lẺp bờng cịc tÝch gièng nhđ bờng cỏu


chđểng vộ so sịnh ệÓ từm nghiỷm. ChỬng hỰn ệÓ
giời phđểng trừnh 3x2 61 thừ hả lẺp cịc giị trỡ tù
nhiến cựa vạ trịi vắi giị trỡ cựa x tẽng dẵn. Khi ệã
ta thÊy 3.42 48, 3.52 75 nến x nỪm giọa 4 vắi
5. Khi ệã lỰi tiạp tôc lẺp bờng tÝch vắi y lÊy cịc giị
trỡ tõ 40 ệạn 50 cựa phđểng trừnh sau khi nhẹn hai
vạ vắi 100 lộ 3y2 6100, vắi x 10y. Cụ tiạp tôc
nhđ vẺy ta sỳ từm ệđĩc giị trỡ gẵn ệóng cựa x.
Khoờng nẽm 300 trđắc Cềng nguyến, nhộ toịn hảc
lẫi lỰc ngđêi Hy LỰp lộ Euclid ệở phịt triÓn mét
phđểng phịp tiạp cẺn viỷc giời phđểng trừnh bẺc
hai bỪng hừnh hảc. Nhọng nghiỷm cựa phđểng
trừnh ệđĩc giời bỪng cịch dùng hừnh, nghiỷm lộ
chiÒu dội cẵn vỳ. Còng nhđ ngđêi Babylon, ềng
còng khềng cã khịi niỷm vÒ phđểng trừnh, hỷ sè.


Euclid (khoờng 325 - 265 trđắc Cềng nguyến)


ậạn thạ kử thụ 7, nhộ toịn hảc ngđêi Ên ậé
Brahmagupta (khoờng 597 - 670) ệở phịt triÓn
phđểng phịp cựa ngđêi Babylon. Viỷc giời
phđểng trừnh bẺc hai trẻ nến gẵn vắi cịch giời hiỷn
chọ viạt thay cho Èn sè, tuy chđa thèng nhÊt trong
cịch sỏ dơng.


Brahmagupta (kho¶ng 597 - 670)


Vừ khềng biạt phịt minh trến cựa Brahmagupta
nến ệạn thạ kử thụ 9, nhộ toịn hảc ngđêi Irớc lộ
Al - Khwarimi (khoờng 780 - 850) ệở ệđa ra mét


phẹn loỰi cịc loỰi khịc nhau cựa phđểng trừnh bẺc
hai vộ phđểng phịp ệÓ giời tõng loỰi cựa mẫi


1. Bừnh phđểng mét Èn sè bỪng Èn sè, tđểng tù
nhđ ngộy nay ta giời phđểng trừnh 2x2 3x.


2.Bừnh phđểng cựa Èn sè bỪng mét sè, 3x2 5.


3.Èn sè b»ng mét sè, 3x 5.


4. Bừnh phđểng cựa Èn sè vắi Èn sè tđểng ệđểng
vắi mét sè, 4x2 3x 8


5.Bừnh phđểng cựa Èn sè vắi mét sè bỪng vắi Èn
sè, 5x2 4 8x


6. Bừnh phđểng cựa Èn sè bỪng vắi vắi Èn sè vộ
mét sè, 2x2 3x 4


SỰ RA ĐỜI CỦA PHƯƠNG TRÌNH



bậc hai, bậc ba và bậc bốn



</div>
<span class='text_page_counter'>(58)</span><div class='page_container' data-page=58>

57


Al - Khwarimi (kho¶ng 780 - 850)


Nẽm 1145, mét cuèn sịch bỪng chọ La tinh cựa
nhộ toịn hảc ngđêi Tẹy Ban Nha gèc Do Thịi
Abraham thanh Hiyya (1070 - 1136) ệở giắi thiỷu
cho ngđêi chẹu ằu nhọng thộnh tùu vÒ giời


phđểng trừnh bẺc hai cựa ngđêi Hăi giịo. Cuèn
sịch ệở cung cÊp cịc phđểng phịp hoộn chửnh ệÓ
giời phđểng trừnh bẺc hai.


Mét giai ệoỰn mắi cựa toịn hảc bớt ệẵu tỰi Italia
khoờng nẽm 1500. Trđắc ệã, nẽm 1494, sịch
Suma cựa nhộ toịn hảc ngđêi Italia lộ Luca
Pacioli ệđĩc in lẵn ệẵu, trong ệã tãm tớt tÊt cờ cịc
thộnh tùu toịn hảc ệở ệỰt ệđĩc cho ệạn thêi ệiÓm
ệã. Trong sịch ệđa ra nhọng kÝ hiỷu biĨu diƠn mét
phđểng trừnh rÊt gẵn vắi cịch kÝ hiỷu ngộy nay.
Tuy sịch khềng bộn ệạn phđểng trừnh bẺc ba
nhđng ệở bộn ệạn phđểng trừnh. Sau ệã, khoờng
nẽm 1515 S. del Ferro (1465 - 1526), mét nhộ
toịn hảc ngđêi Italia ệở ẹm thẵm từm ra cềng thục
nghiỷm ệÓ giời phđểng trừnh bẺc ba. Tuy nhiến vừ
khềng biạt sè ẹm nến ềng ệở khềng cềng bè nã
vộ truyÒn lỰi cho mét hảc trư. Chử ệạn khi biạt ệđĩc
viỷc cã ngđêi ệở biạt cịch giời phđểng trừnh bẺc
ba thừ nhộ toịn hảc ngđêi Italia lộ Nicolo Tartaglia
(1500 - 1557) mắi tiạt lé cềng thục giời cựa mừnh
vắi mét ngđêi bỰn lộ Cardano (1501 - 1557).
Sau ệã Cardano ệở cềng bè cềng trừnh nộy vộ
ngộy nay mải ngđêi vÉn gải ệã lộ cềng thục
Cardano ệÓ giời phđểng trừnh bẺc ba. ậóng ra
phời gải ệẹy lộ cềng thục Tartaglia. Trong cềng
thục nộy, xuÊt hiỷn nhọng sè ẹm cẵn lÊy cẽn bẺc
hai. Cardano ệở từm hiĨu dẵn vỊ loỰi sè nộy mộ
sau nộy ệđĩc gải lộ sè phục. Sau ệã, Cardano
cềng bè trến sịch 20 loỰi phđểng trừnh bẺc bèn



cña mình là Ferrari tiếp tục tìm hiểu.


Tartaglia (1500 - 1557)


Sau nộy Ferrari ệở giời quyạt ệđĩc bội toịn nộy
bỪng cịch chun tõ viỷc phđểng trừnh bẺc bèn vỊ
phđểng trừnh bẺc ba. Ngộy nay, ta gải ệã lộ cềng
thục Ferrari ệÓ giời phđểng trừnh bẺc bèn.


Nẽm 1631, nhộ toịn hảc ngđêi Anh lộ Harriot ệở
dỉng kÝ hiỷu “a a a 5a 12” ệÓ chử mét phđểng
trừnh bẺc ba. Cịch viạt mét phđểng trừnh nhđ ngộy
nay do nhộ toịn hảc ngđêi Phịp Descartes sỏ
chử hỪng sè vộ x, y, z, ... ệÓ chử biạn sè.


</div>
<span class='text_page_counter'>(59)</span><div class='page_container' data-page=59>

58


đên cựa ềng cã cẹy khạ ngảt. TuÊn


lắn lến cỉng vỡ ngảt Êy qua bn tay


chm sóc ca ng.



Vị ngọt càng ngät thªm cïng giäng kĨ của


ông từ câu chuyện cổ tích mà ông cứ gäi lµ



Tói ba gang

cho Tn nhí.



ThÊy Tn hảc bội xong, ềng ệụng dđắi gèc


khạ, gải:



- Nào mang túi ba gang ra đựng khế.




Thạ lộ TuÊn chỰy ra vđên, quÊn quýt bến


ềng. TuÊn vui mét, ềng vui mđêi. Cụ nhừn ịnh


mớt ềng chịu th bit.



Chỉ có điều bố mẹ Tuấn có mỗi mình Tuấn.


Vậy ai là anh, ai là em nh câu chuyện «ng


kĨ...?



Rồi có lần khi

túi ba gang

đã đựng đầy khế,


thấy Tuấn loay hoay, ơng nói:



- Ch¸u mang chia cho c¸c em cïng làng,


cùng xóm.



- Sao lại vậy hả ông? - Tuấn hỏi.



- Thế cháu muốn là anh, hay là em nh trong


câu chuyện?



- Nhng cháu chỉ có một mình...

à

, mà cháu


hiểu ý ông rồi.



Ch một lt, mấy a bn ca TuÊn ệở ệđĩc


thđẻng thục vỡ ngảt cựa nhọng quờ khạ từ tay


Tuấn.



Vị ngọt ấy thấm vào lòng ông...



C

aõy

k

heỏ




</div>
<span class='text_page_counter'>(60)</span><div class='page_container' data-page=60>

59



Ngộy 14.7.2014 tỰi Hộ Néi, Bé trđẻng Bé Giịo
dôc vộ ậộo tỰo GS. TS. PhỰm Vò LuẺn ệở trao
quyạt ệỡnh bữ nhiỷm cịc chục danh lởnh dỰo
Nhộ xuÊt bờn Giịo dôc Viỷt Nam:


GS.TS Vũ Văn Hùng, Phó Tổng Giám đốc kiêm
Tổng biên tập NXBGDVN giữ chức vụ Tổng
Giám đốc kiêm Tổng biên tập NXBGDVN.


ủy viên Hội đồng thành viên
kiêm Giám đốc Tài chính NXBGDVN giữ chức vụ
Phó Tổng Giám đốc NXBGDVN.


ủy viên Hội đồng thành
viên NXBGDVN, Phó Giám đốc NXBGD tại Hà
Nội giữ chức vụ Phó Tổng Giám đốc NXBGDVN.


thuật và Truyền thông giữ chức vụ Phó Tổng
Giám đốc NXBGDVN.


NXBGD tại TP. Cần Thơ giữ chức vụ ủy viên Hội
đồng thành viên NXBGDVN.


TS. Phan Xn Thành, Phó Giám đốc Cơng ty
CP Dịch vụ xuất bản Giáo dục Hà Nội giữ chức
vụủy viên Hội đồng thành viên NXBGDVN.
Tạp chí Tốn Tuổi thơ xin chúc mừng Tổng Giám


đốc, các Phó Tổng Giám đốc và ủy viên HĐTV
NXBGD Việt Nam.


PV


TIN TỨC - HOẠT ĐỘNG - GẶP GỠ



TRÁCH NHIỆM MỚI



nh: Phan Ngảc Quang
Bé trđẻng Bé Giịo dôc & ậộo tỰo GS. TS. PhỰm


Vũ Luận và Chủ tịch HĐTV NXBGD Việt Nam
NGƯT. Ngô Trần i cùng các Tổng Giám đốc, Phó
Tổng Giám đốc và y viên Hội đồng thành viên


Bµi 6NS.


KĨ cịc ệđêng cao BD vộ CE cựa tam giịc ABC.
Ta chụng minh ệđĩc


nến B, E, I, D, C cỉng
thuéc ệđêng trưn ệđêng kÝnh BC.


Suy ra MD ME vµ ID IE.


Mộ nến A, D, K, H, E
cỉng thuéc ệđêng trưn ệđêng kÝnh AH.


Suy ra ND NE vµ KD KE.



Do ệã M, I, N, K cỉng thuéc ệđêng trung trùc cựa
DE.


NhẺn xĐt.Cịc bỰn sau cã lêi giời tèt: NguyÔn Thỡ
Thanh Hđểng, 9A, THCS Yến Phong, Yến Phong,


Bớc Ninh; Ngề Lế Phđểng Trinh, 9E, THCS
Lđểng Thạ Vinh, TP. Tuy Hưa, Phó Yến.


Cịc bỰn sau ệđĩc khen kừ nộy: NguyÔn Thỡ Thanh
Hđểng, 9A, THCS Yến Phong, Yến Phong, Bớc
Ninh; Lế NguyÔn Quúnh Trang, 7C, THCS Vẽn
Lang, TP. Viỷt Trừ, NguyÔn Thỉy Dđểng, 6A3,
THCS Lẹm Thao, Lẹm Thao, Phó Thả; Ngề Lế
Phđểng Trinh, 9E, THCS Lđểng Thạ Vinh, TP. Tuy
Hưa, Phó Yến; Ngun Thỡ HỪng, 8B, THCS Lý
NhẺt Quang, ậề Lđểng, Nghỷ An.


nh cịc bỰn ệđĩc khen ẻ bừa 2.


NguyÔn Ngäc Minh


o
AEH ADH AKH 90


o
BIC BDC BEC 90


</div>
<span class='text_page_counter'>(61)</span><div class='page_container' data-page=61>

60




Mây đen dừng lại trên trời



Để mưa đỗ xuống tưới đồng lúa xanh


Mặt trời tỏa nắng chói chang



Cho nhành lúa chín trĩu vàng thơm bơng


Rạng ngời gương mặt nhà nơng



Nắng, mưa chung sức góp cơng ngày mùa.



CAO NGỌC TOẢN



(GV. THPT Tam Giang, Phong Điền,


Thừa Thiên - Huế)



Một nghìn hai trăm mét


Cao ngun ngàn thơng reo


Vượt tám trăm cây số



Từ Sài Gòn ồn ào


Một thị trấn tĩnh lặng


Trải dài Măng Đen đây


Nhà vẫn ở trong cây



Cây trong rừng xanh thẫm


Người ở những nơi nào


Mà sao nhìn chưa thấy


Đà Lạt thứ hai vậy


Mát rượi suốt đêm ngày



Bõ công vượt đèo núi


Kon Rẫy, Kon Plong


Đến Kon Tum phải biết


Bờ Y và Măng Đen



</div>
<span class='text_page_counter'>(62)</span><div class='page_container' data-page=62>

61



A mother asked sadly her son:



- In all subjects, you got good marks. Why was your behaviour


bad?



Her son answered:



- With the subject of behaviour, I couldn’t look at my friends’


exer-cise to copy.



Hă Thỡ Thu Hờng



(GV. TH Ngô Đức Kế, Can Lộc, Hà Tĩnh)



K ny Chự Vđên ệở cho cịc con vẺt trèn rÊt


kỵ, thạ nhđng dỉ Èn nÊp hạt sục tinh khền,


cịc con vẺt vÉn bỡ cịc bỰn từm ra: BEAR;


CAMEL; CAT; COW; FISH; HORSE.



Chự Vđên sỳ trao quộ cho nhọng bỰn sau:


Nhãm

NguyÔn Thỉy Linh, NguyÔn Bừnh


Dđểng, NguyÔn Hđểng Xen, NguyÔn Huy


Quý, Lđu ậục MỰnh

, 6B;

Dđểng Thỡ Mđêi

,



6A, THCS Lý Tù Trảng, Bừnh Xuyến,

Vỵnh


Phóc

;

ậộm Nam Khịnh

, 7A2, THCS HỰ Hưa,


HỰ Hưa,

Phó Thả

;

Hă Xuẹn Viỷt Anh

, 6A,


THCS Hă Xuẹn Hđểng, Quúnh Lđu,

Nghỷ


An

;

NguyÔn Thỡ Ngảc Diỷp

, 6/5, THCS Lế


Vẽn Thiếm, TP. Hộ Tỵnh,

Hộ Tỵnh

.



Chự Vđên



(TTT2 sè 134)



</div>
<span class='text_page_counter'>(63)</span><div class='page_container' data-page=63>

62



hếm mét ngoỰi ngọ lộ thếm mét cuéc ệêi. Cẹu nãi nữi tiạng Êy nhiÒu
ngđêi biạt. Nhđng Ýt nhÊt nạu chđa cã ệđĩc mét ngoỰi ngọ hoộn chửnh
thừ bỰn cịng cẵn biạt mét chót tiạng Anh ệĨ mẫi khi xt cờnh ệì lóng
tóng. BỰn sỳ phời khai mét tê khai ệÓ nhẺp cờnh vộo cịc nđắc vắi cịc cẹu hái thđêng gẳp:


Full Name in Passport (BLOCK LETTERS) Tên đầy đủ trong Hộ chiếu (CHữ IN HOA)


Passport Number Sè hé chiÕu


Place of Issue N¬i cÊp


Date of Expiry Ngày hết hạn


Date of Birth Ngày sinh


DD - MM - YYYY Hđắng dÉn ệĨ hiĨu viạt vỊ ngộy bỪng 2 con sè,
thịng 2 con sè vộ nẽm 4 con sè



Nationality Quèc tÞch


Identity Card Number Sè chụng minh thđ, sè giÊy cẽn cđắc


Flight No Sè chun bay


Vessel Name Tªn cđa chun tµu biĨn


Vehicle No Sè xe


For office use only Dµnh cho cơ quan (hải quan)
(Không phải khai vào mục này)


Place of Residence Nơi c trú, chỗ ở


Last City/ Port of Embarkation Before X Thộnh phè/ Hời cờng ệở lến tộu trđắc khi ệạn X


Next City/ Port of Disembarkation After X Thành phố/ Hải cảng sẽ xuống tàu sau khi đến X


Address in X Địa chỉ ở X (nơi bạn sẽ n)


Length of Stay Thời gian ở lại


Days Số ngày


Important Notice Chó ý quan träng


Please DO NOT remove this portion §õng rời phần (tờ khai) này



from your passport / travel document khỏi hộ chiếu / giấy tờ du lịch của bạn


i

nc ngoi



</div>
<span class='text_page_counter'>(64)</span><div class='page_container' data-page=64>

63


Hỏi:

Anh Phó ơi! Tại sao 0,(9) 1 ạ?



Một bạn giấu tên


Đáp:



0,(9) 0,999...



Nhiều v tn sè 9 em ộ


Mừnh cụ viạt ệạn tẺn lóc giộ


Cịng chđa hạt nhọng con sè 9


Nến 0,(9) ệđĩc coi lộ 1.



Hái:

Anh Phã thẹn mạn! Em rÊt muèn trẻ


thộnh céng tịc viến cựa Toịn Tuữi thể. Em


ệở gỏi nhiÒu bội răi mộ vÉn chđa ệđĩc ệẽng.


Muèn trẻ thộnh céng tịc viến cựa TTT th em


cn lm g ?



Nguyễn Hà Linh



(7A2, THCS Lê Văn Thiêm, TP. Hà Tĩnh,


Hà Tĩnh)



Đáp:




Em cha nói rõ bài gì


Em cha nói rõ mục gì


Nên anh cha thể nói chi



Em nến gỏi tiạp hay thừ ệđĩc ngay.



Hái:

NÕu mn gưi nhiỊu bµi tham dự giải


toán qua th nhng chỉ có một phiếu đăng kí


thì em phải làm thế nào ạ?



Một bạn ở THCS Đặng Thai Mai, Vinh,


Nghệ An


Đáp:



Bỏ chung trong một phong b×



Bến ngoội dịn phiạu dù thi ệđĩc răi


Thự tơc ệển giờn thạ thềi



Mong em giải đúng, xong xuôi gửi bài.



Hái:

Anh Phó ơi! Tại sao tên báo là Toán Tuổi


thơ ạ?



Mẫn Văn Mạnh



(7A2, THCS Yên Phong, Yên Phong,


Bắc Ninh)



Đáp:




Nội dung thừ lộ toịn


Trừnh ệé tuữi thể thềi


ậèi tđĩng còng tuữi thể


Thạ lộ thộnh tến bịo


Nhđng cã nhiỊu mơc lớm


Em cụ ệảc mộ xem


Dẵn dộ em sỳ quen


Mong bịo vÒ thẺt sm.



</div>
<span class='text_page_counter'>(65)</span><div class='page_container' data-page=65>

64



Bài 1(137+138). So sánh với 3, biÕt


A 1 22014 32013 42012 ... 20142 2015,
B 1 22013 32012 42011 ... 20132 2014.


tèng thµnh vị


(Học viên lớp Cao học Giải tích K5, Đại học Hồng Đức)
Bài 2(137+138). Biết x, y, z là những số nguyên thỏa mãn (x3 y3 z3) 27.
Chứng minh rằng hoặc cả ba số x, y, z cùng chia hết cho 3, hoặc hai trong ba
số đó có tổng chia hết cho 9.


Thịi nhẺt phđĩng


(GV. THCS NguyÔn Vẽn Trẫi, Cam Nghỵa, Cam Ranh, Khịnh Hưa)
Bội 3(137+138).Giời phđểng trừnh:


cao ngảc toờn(GV. THPT Tam Giang, Phong ậiÒn, Thõa Thiến - Huạ)


Bội 4(137+138).Cho a, b vộ c lộ cịc sè thùc dđểng tháa mởn


Chøng minh r»ng


Cao Minh Quang(GV. THPT chuyên Nguyễn Bỉnh Khiêm, Vĩnh Long)
Bài 5(137+138).Cho tập hợp P {đỏ, xanh, đen, trắng}.


Hãy xem mỗi cách chia sau có phải là một phân hoạch của P:
a) P<sub>1</sub> [{đỏ}, {xanh, đen}] b) P<sub>2</sub> [{trắng, đen đỏ, xanh}]
c) P<sub>3</sub> [ , {đỏ, xanh}, {đen, trắng}].


vò kim thñy


1 1 1 <sub>3.</sub>
6a 1 6b 1 6c 1 7


1 1 1 <sub>1.</sub>
2a 1 2b 1 2c 1
2 x 1 <sub>2 x 1</sub> <sub>x</sub> <sub>x 1.</sub>


x x


A
B


Bội 6(137+138). Cho hừnh vỳ vắi ệiĨm O nỪm trong tam giịc ABC
ệỊu. Biạt diỷn tÝch phẵn tề mộu bỪng nỏa diỷn tÝch tam giịc ABC.
Chụng minh rỪng ệiÓm O thuéc mét trong cịc ệđêng trung tuyạn
cựa tam giịc ABC.



nguyễn đễ (Hải Phòng)


</div>
<span class='text_page_counter'>(66)</span><div class='page_container' data-page=66></div>

<!--links-->

×